AAMC Biology question pack 1 difficulty

The #1 social media platform for MCAT advice. The MCAT (Medical College Admission Test) is offered by the AAMC and is a required exam for admission to medical schools in the USA and Canada. /r/MCAT is a place for MCAT practice, questions, discussion, advice, social networking, news, study tips and more. Check out the sidebar for useful resources & intro guides. Post questions, jokes, memes, and discussions.

  • #5

I just finished doing the AAMC biology question pack #1.
Is this the difficulty of the MCAT 2015?
Also, if I got a raw score of 73%, what score would that approximate to on the real test?
Thank you!

@mcatjelly is spot-on. The only thing you might be able to extrapolate a hypothetical score from would be a full length exam or a section test (a full 95 minute or 90 min section on its own). Even then, it would just be a gross estimation, especially given that no one can know the raw-to-scaled score for the new MCAT. It will take multiple AAMC released practice tests with their own scales to be able to get at an average conversion like there was for the old MCAT.

The AAMC science Q packs are all meant to be a way to reinforce science content. It means next to nothing as a predictor of a scaled score on the real thing. No single MCAT section would test all that bio. Where's the biochemistry, the orgo? Just use you % correct as a good indicator of:

1) Are you remembering your science content
2) Can you handle when the MCAT tests that content it its "special" way.

Nothing more.

Right now the best we have is the excellent google doc set up HERE Read it with a grain of salt as this small, mostly unverified data set is in NO WAY statistically meaningful. However it does provide anecdotal evidence as to your possible score given previous performance and that is better than nothing.

Good luck!

Last edited: Jul 10, 2015

MCAT Content / AAMC MCAT Biology Question Pack 1 Solutions

AAMC QPack Bio1 [Web]

Biology Question Pack Volume 1: Passage 1

1) First thing I’m doing is deciding whether I need the information in the passage to answer this question, or whether this is a pseudo-discrete, or standalone question. Even though the passage talked about the sodium pump, I’m leaning toward using my external knowledge of the sodium pump. AAMC loves doing this where they’ll ask you a question that’s tangentially related to the passage, but in reality it’s just a standalone question you can answer using your general knowledge. From our content outline we know that the sodium potassium pump is the most common example of active transport on the MCAT. We have two potassium ions moved into the cell, and three sodium ions moved out of the cell. The pump creates gradients in nerve cells, allowing for electrical signals. The pump is also used in the kidneys to maintain ion balance in the body. It also helps maintain blood pressure, and plays a role in cardiac contractions. Which is to say the pump is obviously important throughout the body. We’re going to try and find an answer that matches one of these main functions.

    1. Veins: Veins return deoxygenated blood to the heart. I mentioned the sodium potassium pump is a player in maintaining blood pressure. I’m thinking this is a viable answer choice, but it’s not as direct as the answers we typically see from AAMC.
    2. Loop of Henle: The loop of Henle is a U-shaped tube that consists of a descending limb and ascending limb. It transfers fluid from the proximal to the distal tubule. The descending limb is highly permeable to water but completely impermeable to ions, causing a large amount of water to be reabsorbed. Sodium and potassium are reabsorbed through the walls of the ascending limb. The walls of the thick ascending limb are impermeable to water. And here, the sodium concentration gradient has to be maintained. The fact that the concentration of urine is regulated so closely here, the loop of Henle is going to be our best answer for the time being. Remember we’re looking for the structure where the sodium potassium pump is most active. That’s why I’m liking answer choice B here.
    3. Lungs: The sodium potassium pump can be necessary to regulate fluid in the lungs. Again, the pump is used to create and maintain that concentration gradient. This is more important when you see a lung malady, or excess fluid. I’m still sticking with answer choice B for “most active” and the best answer so far
    4. Bone Marrow: This is not typically a structure that we associate with active sodium potassium pumps. While we may have the pump present, we’re not expecting the same, active pump that we would in nerves, or in the kidneys. I’m still going to stick with our best answer here, answer choice B. We didn’t end up needing the passage much (or at all) to answer this.

2) In other words, we saw swelling in the passage. This comes down to knowing the reason behind the swelling of the cells when they were placed in distilled water. During your initial readthrough you always want to visualize any experimental setup, especially a visual description like this. If you can’t visualize the experiment, you’re not understanding it clearly.
The passage says Reconstituted erythrocytes are formed by bathing erythrocytes in distilled water under controlled conditions. These cells swell, forming pores that release cytoplasmic proteins and ions into the distilled water. What we’re focused on is the reasoning behind the swelling. We know the erythrocytes contain proteins and ions, meaning there is an initial concentration gradient. Osmosis is the diffusion of water across the membrane. The movement of the distilled water can change the cell’s volume, and that’s what’s happening here. After the cell swells, that’s when the pores form and release cytoplasmic proteins and ions. But that is the result of the cell swelling, not the cause. We want an answer choice consistent with the movement of the distilled water into the erythrocyte.

    1. gradient of ions causes water to enter the cells. Right away, I’m liking this answer choice. When I broke down the question, I said the distilled water will cross the membrane of the erythrocyte due to the concentration gradient. That causes the cell to swell, and eventually open pores which allow for the release of cytoplasmic proteins and ions. 
    2. contractile filaments of the cytosol open pores in the plasma membrane. This is the result of the swelling, not the cause of the swelling that we need to explain to answer this question. Answer choice A is a superior answer choice here.
    3. sodium pump transports sodium out of the erythrocytes more rapidly than normal. This is not the cause of the swelling. Sodium leaving the erythrocyte would have the opposite effect. By increasing the sodium concentration outside of the erythrocyte, less water would enter the cell. 
    4. erythrocyte’s DNA produces degradative enzymes. Degradative enzymes do what their name suggests and degrade molecules. Enough enzyme activity can degrade the membrane, but that’s not what we’re looking for here. Breaking down the membrane is not what causes the cell to swell. We stick with answer choice A. This comes from knowing the experimental setup and using that to answer the question.

3) In simple words, we want to explain whether the sodium pump causes action potentials along neurons. This is going to be similar to question 1 because it’s almost like a standalone, or discrete, question. What do we know about action potentials? Neural impulses occur when a stimulus depolarizes a cell membrane, prompting an action potential which sends an “all or nothing” signal. Transmission of a signal within a neuron is carried out by the opening and closing of voltage-gated ion channels. This, in turn, creates an action potential. Once the threshold potential is reached, the neuron depolarizes and opens the voltage-gated calcium channels. When depolarization is finished, the cell resets its membrane voltage back to resting potential. So how does our sodium-potassium pump fit in here? The sodium-potassium pump maintains the resting potential, once it’s established. It’s not the direct cause of the action potentials along neurons. We’re focused on causation here.

    1. No; action potentials result in an increased permeability of the plasma membrane to sodium. I said the sodium pump isn’t the direct cause of action potentials along neurons. Rather the influx of sodium from the opening of the voltage-gated ion channel cause the depolarization.
    2. No; the myelin sheaths of neurons prevent movement of ions across the plasma membranes of the neurons. Based on the initial breakdown of the question, I do think the answer is “no.” But between myelin sheaths we have the nodes of Ranvier that are not myelinated. There is sodium movement down these unmyelinated nodes. Also, the movement doesn’t cause action potentials.
    3. Yes; sodium is transported out of neurons during action potentials. I said the pump does not cause action potentials. Also, sodium is transported out of the cell by the sodium potassium pump, meaning I’m not a fan of answer choice C.  
    4. Yes; action potentials are accompanied by the hydrolysis of ATP. Again, I’m focused on causation. In the overview, I didn’t say the energy from the hydrolysis of ATP was responsible for the influx of sodium, or causing the action potential. That means we’re left with our best remaining answer choice, answer choice A. 

4) We could reference the equation in the passage, but first we’re going to answer this question using 1) our general knowledge and knowing the number of ATP molecules produced per glucose molecule in glycolysis, and 2) how many sodium ions are transported out of the cell by the sodium potassium pump per molecule of ATP used. The answer is as simple as a net production of 2 ATP, which also means 6 total molecules of sodium.
Alternatively, I can look at reaction 1 from the passage. First thing we want to do is recall the number of ATP molecules produced per glucose molecule in glycolysis. For every glucose molecule that undergoes glycolysis, there’s a net production of 2 ATP molecules and 2 NADH molecules. Look at the reaction, and recall that for a single molecule of ATP, three sodium ions are removed from the cell. If we had two ATP molecules instead of a single molecule, that number would be doubled. That means 6 molecules of Na+.

    1. 3 molecules is an incorrect answer. You would get this if you accidentally used a single molecule of ATP.
    2. 6 molecules match the number of molecules we came up with. 
    3. 9 molecules would be the answer if there were 3 ATP molecules instead of 2.
    4. 12 molecules would be the answer if there were 4 ATP molecules. This answer just came down to knowing the content, and specifically glycolysis.

Biology Question Pack Volume 1: Passage 2

5) We’re focused on the causation of gastric cancer here, and relating to the proliferating cells. The passage mentions there is a correlation between H. pylori and gastric cancer. But now we want to explain why increased proliferation of mucosal cells would lead to gastric cancer. The reason for this should theoretically be specific mutations, or presence of genes that lead to cancer. We’re expecting these mucosal cells to be responsible in some way. 

    1. genetic mutations occur in proliferating germ cells. This answer choice is insinuating genetic mutations in germ cells themselves is causing the gastric cancer. It sounds similar to what I said during my initial breakdown-we needed mutations in proliferating cells. That does sound like cancer. But we’re expecting mutations in the mucosal cells, not germ cells. We want to focus on the proliferating mucosal cells in the stomach, which means we’ll look for a better answer.
    2. genetic mutations occur in proliferating somatic cells. This answer choice is more reasonable than answer choice A and similar to what I said in the prediction. What are somatic cells? They’re the cells that aren’t reproductive cells in a living organism. In this case, if we have mutations in the rapidly reproducing mucosal cells in the stomach, those are somatic cells. That’s exactly what we expect, so this is better than answer choice A already. 
    3. the immune system fails to recognize bacterial antigens. The author mentions in the passage “antibodies rarely eradicate the infection; evidently, this pathogen has developed effective ways to elude host defenses.” We’re also told that infection by the antigens doesn’t always lead to gastric cancer. This isn’t consistent with the breakdown I did. Answer choice B is still superior.
    4. crowded mucosal cells are likely to remain in interphase. This description does not sound like gastric cancer. Cancer is associated with uncontrolled cell division. Answer choice D mentions cells that are not dividing, which isn’t what I’m looking for. That means I’m left with the best, and most correct answer, answer choice B.

6) This is something the author brings up in the passage, and now we’re asked to explain the statement. That means we’re going to have to understand how antibodies typically work, and why they might not work in a particular environment or against a specific antigen.
Antibodies are produced by the immune system to fight antigens. Antigens by definition will induce the production of antibodies. But why might an antibody not be effective? Antibodies are proteins that respond to specific antigens. Just like other proteins, antibodies will undergo conformational changes and not work properly under extreme conditions. The passage mentioned many individuals will develop antibodies. But keep in mind the author talks about the extreme pH in the stomach. Antibodies are typically not as effective in extreme pH or temperatures.

    1. Antibody proteins may be denatured in the harsh environment of the stomach. This is consistent with the breakdown I just went through. The harsh environment of the stomach includes a very low, acidic pH. Antibodies will experience conformational changes in these situations and not work properly. 
    2. Antibodies are not generally effective against bacteria. This goes against our breakdown and also contradicts what we know about antibodies and bacteria. Antibodies are meant to counteract antigens. Answer choice A is still superior.
    3. H. pylori infection may suppress the activity of the immune system. This answer choice goes against what we read in the passage. The author mentions the immune system is still able to develop antibodies, but the antibodies themselves are not effective. Answer choice A is still the best. 
    4. Antibodies are not secreted from host tissues into extracellular spaces. We know antibodies are secreted from tissues into extracellular spaces. This answer is factually incorrect, so answer choice A remains the best answer choice for this question.

7) The author mentions the differences between strains of H. Pylori. That’s going to be the key to answering this question. The passage says “Genetic studies of H. pylori have identified genes that are expressed in different strains of this bacterium. One gene, vacA, encodes a toxin. Expression of another gene, cagA, leads to inflammation and may be related to the genesis of gastric cancer”. So what does this tell us? Not all H. pylori strains may possess the same genes. In fact, we get two examples of individual genes that are found in specific strains. Both strains seem like they can affect humans, but affect them differently. 

    1. attack different hosts. This is inconsistent with what we just mentioned in our breakdown. Different strains can attack the same host. We see different effects in the same host from different strains. 
    2. express different genes. That sounds very similar to our breakdown and the examples the author presents in the passage. Different strains express different genes like vacA and cagA. Answer choice B is the best option so far.
    3. exhibit different degrees of resistance to antibiotics. There’s no evidence given in the passage about different strains having different degrees of resistance to antibiotics. We’ll still keep answer choice B as our superior answer choice.
    4. exist in either developed or developing countries. Just like answer choice C, the passage does not mention anything about different strains of H. pylori existing in either developed OR developing countries. The only thing the passage mentions is the prevalence rate of H. pylori in general. We’re left with the best answer: answer choice B.

8) This involves going back to the passage and noting the effect of seeing the cagA gene in our H. pylori strain. The passage says says “Expression of another gene, cagA, leads to inflammation and may be related to the genesis of gastric cancer”. So our answer is going to deal with this inflammation, and possible with the beginning of gastric cancer as well. When an inflammatory response is triggered, usually white blood cells will fight any infection or debris. So we’re expecting inflammation followed by a defensive response.

    1. the disruption of host cell enzymatic activity. We’re not expecting a disruption per se, rather we’re expecting an inflammatory response. It’s a fairly straightforward question and a specific answer, so I’m hoping for something a little better.
    2. the disruption of host cell protein synthesis. Again, we’re not expecting a disruption. We want an answer choice that explains what would happen as an inflammatory response. Again, not ideal so we’ll keep looking for a better answer.
    3. the movement of leukocytes into mucosal tissue. This is consistent with the passage and my breakdown. Expression of the gene leads to inflammation in the stomach cells. Leukocytes, or white blood cells, will head to the site of the injury as an inflammatory response. So we’re expecting the presence of the cagA gene to be followed by the movement of leukocytes into the mucosal tissue. Best answer so far, but I still want to look at answer choice D to make sure.
    4. the vasoconstriction of arterioles in the mucosal layer. When we have inflammation, blood vessels will dilate to increase the flow of blood and immune cells to the site. Answer choice D not a good option. I’m going to stick to answer choice C.

9) The author says in the passage “There is a relationship between H. pylori infection and cancer.” But then immediately after says “Infected individuals have a two-fold increased risk of gastric cancer, although >75% of patients with active infections do not develop cancer.” This essentially means less than a quarter of patients with active infections do develop cancer. Most patients have been infected, but they do not actually develop cancer.

    1. do not incorporate bacterial genes in their chromosomes. Make sure to read these answer choices slowly. Note the question says “most do not develop” and now option A says “do not incorporate.” That means the people that are infected by H. pylori won’t develop cancer because bacterial genes don’t incorporate in their chromosomes. The verbiage can get tricky. There’s no mention of this in the passage, but the answer doesn’t explicitly go against the passage either. Let’s see if we can find anything better. 
    2. have robust immune systems that defeat early cancers. Even if a patient has a robust immune system, the author mentions the pathogen has developed effective ways to elude host defenses. This contradicts what the author mentions in the passage, and what I’ve provided in my breakdown. 
    3. eradicate the infection before any tumors develop. This contradicts what we just read from the passage. The passage explicitly says many individuals develop antibodies against H. pylori antigens, these antibodies rarely eradicate the infection. The only mention of eradication in the passage is due to using antibiotics. 
    4. tolerate the infection without developing tumors. This is consistent with our prediction. The passage explicitly says more than 75% of patients with active infections don’t develop cancer. That means out of 100 people that are infected, less than a quarter, or fewer than 25 would actually develop gastric cancer. It also says that antibodies are rarely fully effective to eradicate the disease. Meaning the rest all tolerate the infection and may have adverse effects, but these adverse effects don’t include cancer. This is a superior answer to answer choice A. B and C we already ruled out. We can say most people don’t develop gastric cancer because they tolerate the infection without developing tumors.

Biology Question Pack Volume 1: Passage 3

10) I want to go back to the passage to find a possible cause of cellular dehydration that’s related to hyperglycemia. The passage says “hyperglycemia (high glucose concentration) may protect tissues and organs by depressing the freezing point of body fluids, thereby facilitating the dehydration of cells and other secondary cryoprotective mechanisms”. So, what is this high glucose concentration doing to promote cellular dehydration? The high glucose concentration is to protect the tissues and organs of the frogs by depressing the freezing point of body fluids. Said differently, that means the freezing point of the fluid is lowered by the increased glucose concentration. What this also means, is the concentration of glucose in the fluid is higher than that of the tissues and tissue cells. That means osmosis. Water is going to leave the tissue cells to maintain the proper concentration gradient. That also means the water that leaves the cells and freezes will be extracellular. 

    1. glucose, as an energy source, accelerates the osmotic work performed by plasma membranes. This answer choice tangentially mentions osmosis, but we’re not using glucose as an energy source to drive osmosis to do work. Osmosis can be used to do work and even generate electricity. But that’s not what’s happening here. At best, we have the right components, and some of the right thinking in this answer choice. But we want to keep looking at other answers, which we should be doing anyway.
    2. Answer choice B says glucose, as an energy source, accelerates plasma membrane ion exchange pumps. Just like our last answer choice, glucose isn’t being used as an energy source. Osmosis does not require ion-exchange pumps. Water can simply move across the plasma membrane from intracellular to extracellular. This contradicts what I said in the initial breakdown. I’m liking answer choice A still. 
    3. Answer choice C says glucose molecules raise the osmotic pressure of the extracellular space. This is exactly what the passage mentions happens. The example the author gives is the hepatic glycogen stores are released and this increases the glucose concentration in the body fluid. Water crosses the membrane from intracellular to extracellular space, and that dehydrates the cell. This is better than answer choice A, and we already ruled out answer choice B.
    4. Answer choice D says glucose molecules are exchanged for water molecules across the plasma membrane. Glucose does not enter the cells. Water leaves the cell across the membrane, but not through an exchange with glucose. That means answer choice C is the best option.

11) This means we’ll use our knowledge of normal glucose regulation, and compare to what’s going on in these species of animals. We already touched on this in our previous questions, but hyperglycemia means high glucose concentration. Normally, the body adjusts for this elevated glucose concentration. Excess glucose in the blood is converted into glycogen and is stored in the liver or muscles through glycogenesis. The insulin hormone stimulates this synthesis of glycogen. When blood glucose levels rise, insulin stimulates glycogen synthase to form glycogen, from glucose. Glucagon-acts opposite to insulin. Glucagon stimulates the breakdown of glycogen whenever blood glucose level falls. So in this case, we’d expect the elevated blood glucose levels would cause insulin to stimulate glycogen synthase, and form glycogen. 

    1. Suppression of insulin secretion during freezing episodes. This matches what I said in the breakdown of the question. I said insulin secretion would normally be expected in this situation. There’s elevated glucose, so insulin wants to stimulate glycogen synthase and form glycogen. That’s not what’s happening. That insulin activity is being suppressed. 
    2. Suppression of glucagon secretion during freezing episodes. This is the opposite of what I said during the breakdown of the question. I said glucagon stimulates the breakdown of glycogen when blood glucose levels fall. Blood glucose levels are currently higher, so we’re not expecting glucagon levels to be very high anyway. Answer choice A is still our best option. 
    3. Slowing of glycogen catabolism in the liver during freezing episodes. This answer choice sounds like a normal glucose regulatory mechanism. Slowing the breakdown of glycogen in the liver would be expected when there’s elevated blood glucose already. This answer choice is unreasonable, so I’ll stick with answer choice A as the best answer so far.
    4. Increased sensitivity of all pancreatic endocrine responses during freezing episodes. Changing the sensitivity of pancreatic endocrine responses would theoretically cause more insulin to be secreted. This is not a change in the actual regulatory mechanism. And because we’re seeing the opposite in terms of insulin, this answer contradicts what we expect to happen. Answer choice A wins out.

12) The fact that the question stem mentions we should look at Figure 1 is probably a good reason to flip back to the passage. But theoretically, we should be able to use general knowledge to answer this question. Just because a Table, Figure, or Molecule is referenced in a question stem, doesn’t always mean it’s necessary to flip back to the passage.

When looking at Figure 1, the vertical dotted lines represents the onset of freezing and thawing in all 5 responses. And those happen at 0 and 24 hours respectively. So we want to know in our second chart (the “heart rate” chart), why is there still a pulse for several hours after the onset of ice formation. And how does this help glucose carry out its role. What is glucose’s role? We were told in the passage that elevated glucose may protect tissues and organs by depressing the freezing point of body fluids, thereby facilitating the dehydration of cells and other secondary cryoprotective mechanisms. Glucose is essentially circulating through the body, and allowing for dehydration of cells throughout the body. How does that work? The presence of this elevated glucose concentration leads to osmosis, and water moving extracellularly.

    1. circulating blood distributes the glucose throughout the body tissues. Does this answer the question? Yes, it does. It mentions what’s happening with the pulse, and it’s also addressing glucose’s key role. Glucose has to be distributed throughout the body for the necessary osmosis and dehydration to happen. Initial impression is a good one. Let’s see if we can find anything better.
    2. circulating blood equilibrates the temperature throughout the body. While this could be true, we have to be careful. How does circulating blood equilibrating the temperature in the body affect glucose? This answer doesn’t exactly answer the specific question being asked. Remember to always pick answer choices that consider the scope of the specific question being asked. We like answer choice A for now.
    3. a beating heart warms body tissues and slows ice formation. Again, this is another answer that doesn’t quite answer the question being asked. There’s no mention of how this pulse is crucial to the role of glucose. Even if the answer is true, it is outside the scope of the question.
    4. a beating heart requires a constant supply of glucose as an energy source. Another answer choice that’s technically correct, but it doesn’t answer the specific question being asked. Another answer that is out of scope, so we’re picking answer choice A for answering the specific question being asked. Whenever you have the ability to eliminate answer choices for being out of scope, make sure to do so.

13) Keep in mind we can pick more than 1 option here. Even though the question stem says “according to the passage,” there’s really no need to go back to our passage here, and that’s perfectly fine. The goal is to read the passage in such a way that you understand the big picture and all of the moving parts. It’s not always mandatory to go back to the passage, even if a question says something like “according to the passage.” 

There is dehydration of cells, and we expect ice in the extracellular fluid. That’s all this question is asking us-which of our answer choices is extracellular? Blood plasma and lymph. Which of our answer choices is intracellular? Cytoplasm. 

The author mentions “Formation of ice within cells disrupts structural organization and metabolic function, and ultimately causes cell death.” If we have frozen cytoplasm, we have a lot of big issues. That means our correct answer choices are options 2 and 3, because option 1 would be catastrophic. Answer choice D contains the combination we’re looking for: II and III only. All of the other answer choices can be eliminated for being incorrect.

14) We essentially want to know when the frog’s heartbeat is at zero, and it’s using anaerobic respiration. We can look at Figure 1 in the passage, and I already said anaerobic respiration was when the frog’s heartbeat was at zero. What does the heartbeat being at zero entail? It means the heart isn’t circulating oxygenated blood. And anaerobic respiration happens when oxygen is absent. 

Look at Figure 1, and we’re focusing on heart rate, because that’s going to tell us when there’s oxygenated blood circulating or not. When do we have our beats/minute equal to zero? It looks like from hour 12, to a bit past hour 24. Maybe around 25 or 26 hours.

    1. Before 0 hours. This is an incorrect value, there was a beat before hour 0 and until hour 12. We’re looking for beats/minute equal to zero.
    2. Answer choice B says Between 0 and 6 hours. This is also incorrect, there was a beat between hour 0 and until hour 12. Meaning the entirety of answer choice B. Answer choice B is less wrong than answer choice A.
    3. Answer choice C says Between 0 and 24 hours. Another incorrect value, but this is better than answer choice B. Half of answer choice C is a time period when the frog is completely reliant on anaerobic respiration. The other half we have a heartbeat. That means answer choice C is better than answer choice B.
    4. Answer choice D says Between 12 and 26 hours. This matches what I said in the breakdown exactly.

15) Top bar graph says survival percentage; bottom bar graph says plasma hemoglobin. We have three different test groups. We have one group which was injected with saline only, and it’s the bars shown on the left side of the two graphs. In the middle, we had saline, and 650 millimoles/liter of glucose. On the right, we had saline and 1500 millimoles/liter of glucose. Top graph we see that the saline only frogs did not survive. The frogs with the lesser glucose concentration had a roughly 40% survival rate. And the frogs with the higher glucose concentration had a roughly 80% survival rate. This injection of glucose increases the survival rate of the frogs in freezing temperatures. But, we want to know about the cryoprotective role specifically, not just the survival rate.

Bottom bar graph shows plasma hemoglobin. We want as little hemoglobin as possible in the plasma. That’s because hemoglobin is carried in red blood cells. If we have elevated hemoglobin levels in the plasma, that means the red blood cells ruptured or were damaged. There’s more plasma hemoglobin in the “saline only” group. Followed by the “650 millimole/liter glucose” group.” And there’s very little plasma hemoglobin in our last group with the highest exogenous glucose levels. A higher level of exogenous glucose leads to a higher survival rate. It correlates to a lower concentration of plasma hemoglobin. And there’s less cell death in the group with the highest exogenous glucose levels.

    1. Yes; survival and protection against hemolysis are promoted by exogenous glucose. What’s hemolysis? It’s the rupturing of red blood cells. That’s exactly what the exogenous glucose is preventing. That matches what I said during the breakdown of the question, so I’m liking this option for now.
    2. Yes; death caused by freezing is directly proportional with the extent of hemolysis. We may be able to say this answer choice is factually correct, but does it address the specific question being asked? There’s no mention of glucose or supporting the hypothesis that glucose has a cryoprotective role. Answer choice B is out of scope, so A is the superior answer here. 
    3. No; injected glucose lowered blood hemoglobin levels, suggesting that the survival rates are not related to the treatment. This answer is insinuating the glucose caused lower plasma hemoglobin levels. In reality, a lack of glucose caused higher blood hemoglobin levels. We have to be careful with causation and correlation. This answer choice is unreasonable. I’m still liking answer choice A.
    4. No; injected saline promoted hemolysis, suggesting death was most likely related to circulatory collapse. Another answer where we have to consider causation, not correlation. The cause of the cell death was a lack of glucose, which I covered extensively. It wasn’t because of the injected saline, so answer choice D is unreasonable. Answer choice A is the correct answer. 

Biology Question Pack Volume 1: Questions 16-20

16) We’re going to consider when water is lost through the skin, or in other words, perspiration, or sweating. 

Sweating, or perspiration, is a method of evaporative cooling. Why does this happen? Thermoregulation, or hormonal changes. Sweating is the production of fluids secreted by the sweat glands in the skin of mammals. Humans have two types of sweat glands: eccrine glands and apocrine glands. Eccrine glands are the major sweat glands of the human body, found in virtually all skin. They produce a clear, odorless substance, sweat, that’s primarily water and sodium chloride. Sodium chloride is reabsorbed in the duct to reduce salt loss. Eccrine glands are active in thermoregulation and are stimulated by the sympathetic nervous system. 

Apocrine sweat glands are inactive until they’re stimulated by hormonal changes in puberty. Apocrine sweat glands are mainly thought to function as olfactory pheromones. The stimulus for the secretion of apocrine sweat glands is adrenaline. So, we’re looking for an answer mentioning either increasing the need for evaporative cooling, or adrenaline stimulating secretion of apocrine sweat glands.

    1. inhibiting kidney function. This is not related to our breakdown, but is this a possible answer choice? Inhibiting kidney function could mean increased urinary output, but does that contribute to the amount of water lost through the skin? It does not. 
    2. decreasing salt consumption. This answer choice ties into answer choice A. Decreasing salt consumption would mean increased urinary output as well. No need for as much water resorption to maintain the body’s proper electrolyte concentration. Another answer choice where urinary output would increase, but we don’t have the same potential for a big difference in water lost through the skin. Both A and B would affect urinary output, so we’ll keep both for now, let’s keep comparing.
    3. increasing water consumption. Three straight answer choices where we’re dealing with increased urinary output, but not necessarily increased water lost through the skin. Unless you’re dehydrated, increasing water consumption isn’t going to have a significant effect on water lost through the skin. We’re going to hold on to all 3 answer choices so far. 
    4. raising the environmental temperature. This answer choice is consistent with our breakdown. We said sweating is a method of evaporative cooling. When external temperature gets too high and the body needs to be cooled down, water is lost through the skin. That water evaporates, and cools down the body. We can eliminate answer choices A-C because those were all unreasonable. None of them significantly affected water lost through the skin specifically. We’re left with our best answer, our correct answer: Answer choice D-Raising the environmental temperature. 

17) In other words, what can we say about lipases because they can catalyze the hydrolysis of fats and other carboxylic acid esters? We’re focusing on the fact the test-maker explicitly tells us about lipases catalyzing the hydrolysis of both fats and carboxylic acid esters.

Gastric lipase is the most common lipase we encounter on the MCAT. It’s secreted by chief cells in the stomach, and it helps in the digestion of lipids. Lipase is a type of esterase, which means it splits esters into an acid and an alcohol through hydrolysis. We can get into some enzyme specificity here as well. Enzymes are suited to bind to a particular substrate to help catalyze a biochemical reaction. In this case, the author brings up different molecules that will undergo the same hydrolysis. Main takeaway is lipases catalyze hydrolysis of multiple molecules. Ultimately, same reaction type, but not just one substrate only.

    1. some enzymes are molecules other than proteins. This answer choice is out of scope. Lipases catalyzing the hydrolysis of both fats and other carboxylic acid esters does not have anything to do with enzymes not being proteins. If we were dealing with ribozymes, or another possible example of answer choice A, this would be a more viable answer choice.
    2. most enzymes interact with only one specific substrate molecule. This is the opposite of what the author demonstrates through the example in the question stem. We’re explicitly told a type of enzyme that interacts with multiple substrate molecules. There’s no mention about enzymes interacting with only one specific substrate molecule. We have to be careful here. Answer choices A and B are technically true, but do they answer the specific question being asked? Don’t get tripped up by correct statements that don’t actually answer the question being asked. 
    3. some enzymes interact with several different substrate molecules that have similar chemical linkages. This sounds like exactly what the author is demonstrating through the lipase example. Even though we have hydrolysis in both situations, there’s interaction with different substrate molecules. We know these molecules have similar chemical linkages-we talked about splitting esters in our breakdown. Answer choice C is superior to answer choices A and B. Neither of those addressed the specific question being asked-they were out of scope.
    4. some enzymes interact with many biologically active substrate molecules of dissimilar structures and linkages. This answer choice is not illustrated by lipases. Even if this were a true statement, lipase catalyzes hydrolysis of different substrate molecules. We’re not seeing different structures and linkages, rather they’re similar like we broke down in our prediction. We can also eliminate answer choice D because it contradicted what the author set up in the question stem. We’re left with our correct answer: Answer choice C: some enzymes interact with several different substrate molecules that have similar chemical linkages

18) We can actually use a visual to help us answer this question.

AAMC Biology question pack 1 difficulty

The F factor plasmid contains genes that allow the plasmids DNA to be transferred between cells. The F stands for fertility-it allows genes to be transferred from one bacterium carrying the factor, to another bacterium that previously lacked the factor. That’s done through conjugation. Conjugation’s the transfer of genetic material between two bacterial cells in direct contact. We have a breakdown of conjugation above. If you need to brush up on this material, or want to look closer at the image, I implore you to check out the science content outline on our website.

    1. Transformation. In transformation, a bacterium will take up a piece of DNA from its environment, often from a degraded bacterium. While the bacterium acquires new genes, the process does not depend on the F factor plasmid. 
    2. Transduction. Transduction is another way bacterium can acquire new genes, but by a bacteriophage or virus, so no plasmid once again. The bacteriophage will inject foreign DNA into a host bacterium. Again, we have new genes, but not dependent on the F factor plasmid. 
    3. Conjugation. This answer choice matches our breakdown and the visual. We mentioned the F factor plasmid contains genes that allow plasmids DNA to be transferred between cells. That process is ultimately conjugation. Answer choice C is superior to both answer choices A and B.
    4. Translocation. Translocation is typically related to movement. For our purposes, and for the purposes of this specific question: translocation can describe a type of DNA mutation where a stretch of DNA breaks off and attaches elsewhere. That’s not exactly what’s happening in the visual, or our breakdown. We’re left with our correct answer, answer choice C: Conjugation. 

19) I want to emphasize the word “some” in the question stem. What makes that word so important? Not all of the offspring displayed these recessive traits; some still displayed the dominant traits. The only way we can have both dominant and recessive phenotypes, is if the offspring inherited a recessive allele for each gene from both parents. What that means is, at least one parent has one dominant allele for both genes-meaning heterozygous. How do we know that? Because we have a mix of dominant and recessive phenotypes. The recessive allele for both traits has to be inherited from both parents. That means one parent has to be VvEe, and the other parent has a few options. The 2nd parent can either be homozygous recessive (vvee), heterozygous for one trait and homozygous recessive for the second (Vvee OR vvEe), or Heterozygous for both (VvEe). That’s the only way to get the experimental results. 

    1. One VVEE, one vvee. This combination of individuals would produce only heterozygous offspring. Meaning VvEe. All offspring would display dominant traits only.
    2. Both VvEe. This answer choice matches our breakdown. It’s possible for offspring to display both recessive and dominant phenotypes for these traits. We can keep this answer choice for now. We can eliminate answer choice A because it contradicted our breakdown and what we need from our correct answer.
    3. One vvEE, one VVee. So both individuals are homozygous dominant for one trait, and homozygous recessive for the other trait.  This answer choice is similar to answer choice A. This combination of individuals would produce only heterozygous offspring. Meaning VvEe. All offspring would display dominant traits only. We can eliminate answer choice C.
    4. Both vvee. This answer shows both individuals as homozygous recessive. This combination of individuals would only produce homozygous recessive offspring. We’re told in the question stem some, but not all, of the offspring show the recessive traits. That means the rest showed dominant traits. That means we can eliminate this answer choice as well. There would be no dominant traits with this combination. We’re left with our correct answer, answer choice B.

20) Contraction of the diaphragm and intercostal muscles causes the lungs to expand. How does that happen? We create a negative intrapleural pressure when the diaphragm contracts. That negative pressure causes the lungs to expand. We have higher volume, which is consistent with Boyle’s Law. That negative pressure causes air from the atmosphere to passively come into the lungs.

AAMC Biology question pack 1 difficulty

    1. diffusion of gases. Diffusion of gases is involved in air entering the lungs, and the movement of oxygen and carbon dioxide. But it’s not responsible for the actual inflation of the lungs. The inflation of the lungs is caused by the negative intrapleural pressure when the diaphragm contracts. This answer choice is a decent one for the time being, let’s keep comparing and see if we can find anything better.
    2. active transport of gases. We don’t have air moving against its energy gradient, which would be the case in active transport. We just mentioned in our previous sentence, and we touched on this in our readthrough: air will passively come into the lung from the atmosphere because of the pressure difference. Active transport is not what causes the lungs to inflate. Answer choice A is superior.
    3. positive pressure pumping action. Positive pressure ventilation is not something we see in mammals, but we see it more in amphibians where they will actually force air down to breathe properly. It’s the opposite of how mammals like humans will breathe, unless we’re hooked up to a ventilator, or we’re having air forced into the lungs. Answer choice A remains superior.
    4. negative pressure pumping action. This answer choice is consistent with our breakdown. Mammals will use negative pressure pumping action. In humans, the diaphragm contracts, the lungs expand, pressure in the lungs drops relative to atmospheric pressure, and air comes into the lungs. This is now the best answer choice, even compared to answer choice A. This answer choice addresses the actual question being asked by the test-maker. That means we can eliminate answer choice A and we’re left with our correct answer, answer choice D: negative pressure pumping action

Biology Question Pack Volume 1: Passage 4

21) There was no explicit mention of rRNA in the passage, so we’re going to use our general knowledge about genes and rRNA to answer this question. Extrachromosomal means these genes are found off the chromosomes. We know chromosomes are replicated and passed on to daughter cells. If these rRNA genes are not found on chromosomes, then they end up replicating by themselves to pass from generation to generation.

    1. nonlinear. We know we’re dealing with protozoa, which are eukaryotic. That means DNA is linear. Upon first consideration, this answer contradicts our passage and what we know about eukaryotes. 
    2. nonfunctional. We know the genes have to be functional to properly carry out protein synthesis. That’s essentially the job of these genes and isn’t completely shut down. We’re not dealing with junk rRNA, so let’s move on to answer C and see if we can get a better answer. 
    3. self-replicating. These genes are not found in chromosomes, so the way they’re passed on is through self-replication. We can keep this answer for now because it’s a better answer than answer choices A and B.
    4. rearranged. There’s no mention in the passage, or in our prediction about these genes being rearranged. This is out of scope within the context of the passage and the question. Best answer here is going to be answer choice C.

22) This boils down to explaining why there’s an extra S phase seen in small macronuclei before division. The passage says “To minimize fluctuations in DNA content, small macronuclei undergo an additional S phase before division, and large macronuclei eliminate an S phase.” Smaller macronuclei will undergo an additional S phase. What normally happens in the S Phase? That’s the synthesis phase, where replication takes place. We have additional DNA replication in small macronuclei.

But no S phase in larger macronuclei. So the larger nuclei have no need for replication. What did we say was the difference? The smaller macronuclei need additional DNA to properly execute amitotic division. The larger macronuclei already have enough.

    1. low concentrations of DNA in the macronucleus. I said during the breakdown of the question that there are low concentrations of DNA in the small macronuclei, so we see an additional S-phase. Answer choice A is a good option for the time being. Let’s look at the rest of the options to see if we can get anything better. 
    2. centromeres in the macronucleus. Centromeres are essential in cell division in mitosis, but that isn’t relevant here. We’ve got amitotic division, not mitosis so this isn’t a good option.
    3. high concentrations of DNA in the micronucleus. This answer is out of scope. This part of the passage, and the question stem are focused on macronuclei, not micronuclei. We can also get rid of option C. 
    4. mitotic enzymes in the micronucleus. This answer contradicts our prediction. We said there is amitotic division, not mitosis. Correct answer here is going to be answer choice A.

23) First thing we want to note is that cytoplasm is distributed unequally during oogenesis. The zygote that results from fertilization needs a majority of the cytoplasm. One mature ovum is formed, and we have three polar bodies. What do we know about macronuclei? We actually mentioned in our last question as well, there’s uneven distribution of macronuclear DNA during amitotic cell division. 

    1. undergo uneven division. This is consistent with my breakdown of the question. I mentioned there’s uneven division of macronuclear DNA. We also know the cell that becomes the ovum gets a majority of cytoplasm, but polar bodies get very little. This sounds like a decent answer and no reasons to eliminate this answer choice right away. Let’s see if we get a superior answer choice. 
    2. contain uneven amounts of nuclear material. This answer choice is a little tricky because there’s uneven amounts of DNA and cytoplasm, but not of nuclear material. If you’re going through the passage, the difference was the amount of DNA, not nuclear material, so I’m liking answer choice A the best still.
    3. regulate their contents by adding or skipping an S phase. This answer choice is great at describing what happens with the macronuclei. We’ve actually seen it in our past few questions. But does this happen with vertebrate ova also? It does not. So this answer choice only applies to half of the question stem. It goes to show how important it is to always consider the question being asked and making sure you address every part of the question when necessary.
    4. are apportioned at mitosis. We know division is amitotic in the macronucleus, so this is factually incorrect. Right away we can eliminate this answer choice. Answer choice A is going to be the best answer.  

24) This answer is going to come from the passage and what we’re told about the macronuclei and binary fission. First thing we want to note from the passage is the macronucleus is 45-ploid. The suffix “ploid” is telling us the number of sets of chromosomes in a cell. We have a 45-ploid macronucleus which we’re told is the site of gene expression. Question stem says a heterozygous macronucleus undergoes repeated binary fission. The last paragraph in the passage tells us about binary fission and each daughter cell receiving an uneven amount of DNA. This amitotic division isn’t something we cover on our content outline. But we are told there’s an uneven amount of DNA.

    1. the loss of macronuclear chromosomes. Even though we have an uneven division, there’s not going to be a loss of chromosomes. A loss of chromosomes might happen if happen if one half of the DNA was unused, but that’s not the case. Not a good start with answer choice A, so we’re still looking for a better answer. 
    2. an increased rate of crossing over in the macronucleus. This answer is out of scope. We’re not seeing crossing over in the macronucleus. Crossing over can happen in meiosis between homologous chromosomes. There’s uneven division, not crossing over. A and B are both not what we’re looking for.
    3. the production of a macronucleus with a genetic origin distinct from the micronucleus. This is inconsistent with the conjugation steps in the passage. The macronucleus arises from the micronucleus in step 6. This contradicts the image in our passage. We’ll see if answer choice D is better, or if we’re picking from which of the 4 sounds best.
    4. a variable allele distribution in the macronucleus. This answer choice looks promising. We said amitotic division leads to an uneven distribution of chromosomes, and uneven distribution of alleles. We can keep answer choice D as the superior answer to answer choices A-C.

25) This is an open-ended question. Sometimes we can predict what the answers going to be, but for this question we’ll break down the question stem and ultimately find the best answer by going through our 4 choices. We’re going to look at Figure 1, Step 6 and see whether the eliminated DNA sequences can be involved in transcription, translation, meiosis, or ribosome production. 

AAMC Biology question pack 1 difficulty

We have step 6 from our passage here, and our possible answer choices on the right. Let’s stick to what we know. Macronuclei will usually control metabolic and developmental functions. Essentially all somatic functions. We’re essentially eliminating DNA sequences that wouldn’t contribute to this purpose. It’s as simple as that. Glancing at our answer choices, we know transcription, translation, and ribosome production are all essential for protein synthesis. We can eliminate meiosis as the odd answer out. We group transcription, translation, and ribosome production together, and said the DNA sequences are not involved in meiosis. Answer choice C is going to be our best answer choice here. 

26) A few things we want to note. The micronucleus is typically for reproduction, and has genetic information. Macronucleus is the site of gene expression and will control all somatic functions. We also know that during conjunction the macronucleus arises from the micronucleus in step 6. So we’re essentially only focusing on the micronucleus to answer this question. Let’s draw out our punnett square to predict our percentage that’s recessive. We’ll call the dominant allele capital A, and the recessive allele lower case a. We fill out our square. 

AAMC Biology question pack 1 difficulty

We have a homozygous dominant, two heterozygous where we’ll observe the dominant allele, and one double recessive. So our percentage is going to be 25%.

We can quickly glance at our numbers and see they’re not particularly close to one another, and our predicted value matches answer choice B.

Answer choice C corresponds to the Aa genotype. Answer choices A and D would not be possible for any genotype.  

Biology Question Pack Volume 1: Passage 5

27) The passage says “The antisense molecules will bind specifically to the sense mRNA and prevent the production of the natural gene product.” The author tells us the antisense molecules which are complementary to portions of the sense mRNA, bind to that sense mRNA. Why do the antisense molecules bind the sense mRNA? To prevent the production of the natural gene product. Said differently, to prevent translation. What is translation? Translation is the process by which a protein is synthesized from mRNA. Without the input of that messenger RNA template translation can’t take place as normal.

    1. DNA replication. DNA replication is the process by which DNA is copied. That usually happens before a cell divides. That’s not what we’re preventing here. We’re focused on mRNA, which means we’re looking for an answer that has to do with translation and not replication. Let’s keep going through our answer choices. 
    2. RNA transcription. Transcription is the process by which mRNA is synthesized from DNA. This would make the drugs just as effective, but this isn’t what’s happening in the passage. The antisense molecules bind this mRNA and keep it from translation. This answer is more relevant and closer to what’s being asked in our question stem. We can keep this answer as the superior answer over answer choice A.
    3. RNA translation. This matches our prediction. We said antisense molecules will bind specifically to the sense mRNA. That prevents the production of the natural gene product. Proteins can’t be synthesized as normal. We’ll keep answer choice C for now; answer C is superior to answer choice B.  
    4. cell replication. We’re not so much focused on preventing replication through these antisense drugs. Remember, the passage specifically tells us the mechanism by which the antisense molecules work, and that’s by preventing RNA translation. Answer choice C is the best answer here.

28) We’re going to explain how the antisense drugs work, and how the gene works as well. What’s the target gene? It’s that harmful gene that we don’t want expressed. The antisense RNA is meant to bind the sense mRNA and prevent translation, like we went over in our previous question. We want the antisense RNA to be expressed any time the target mRNA is expressed.

    1. on the same chromosome as the target gene but not necessarily be physically adjacent. This answer choice focuses more on location. Not necessarily having the antisense RNA and sense mRNA expressed at the same time. Even if the antisense and target gene are nearby, we still want both to be expressed simultaneously to properly prevent translation of the target gene. Let’s keep comparing.
    2. on the same chromosome as the target gene and must be physically adjacent. Again, this answer choice focuses more on location. Not necessarily having the antisense RNA and sense mRNA expressed at the same time. This doesn’t prevent translation. This question is more extreme than answer choice A, so we can keep our broader answer choice for now: answer choice A.
    3. regulated in a similar manner as the target gene. This answer choice is expressing what we want from the antisense gene. We need the antisense gene to be transcribed whenever the target gene’s mRNA is transcribed. Both should be expressed simultaneously to prevent that harmful translation. We can keep answer choice C for now and get rid of answer choice A.
    4. coded on the same strand of DNA as the target gene. Another answer choice that focuses on location. We’re more worried about timing and expression than we are about location. This answer choice isn’t answering the specific question being asked here. Answer choice C is the superior answer.

29) Keep in mind, the disorder is caused by a mutation in the gene for an enzyme. We’re told PKU is caused by a mutation in a gene that eliminates enzymatic activity of phenylalanine hydroxylase. What does that mean? We don’t have a properly functioning enzyme to break down the amino acid phenylalanine. Would an antisense drug help individuals with PKU? Normally The antisense molecules bind the sense mRNA and prevent translation. Unfortunately, preventing the production of the enzyme would not fix the fact that the enzyme itself is not working correctly. If we inhibit the production of the enzyme, we still are unable to break down phenylalanine. We need a remedy for the disordered enzymes themselves, not prevent the enzymes from being made.

    1. Yes, if it binds to the mRNA of the phenylalanine hydroxylase gene and prevents its translation. We said we don’t expect an antisense drug to help with these individuals, but let’s see the reasoning. This answer is saying we inhibit the gene that’s already producing defective enzymes. Preventing translation is not going to fix the root cause of the problem. The defective enzymes themselves. Let’s keep comparing. 
    2. Yes, if it is incorporated into the chromosome and prevents the expression of the phenylalanine hydroxylase gene. This answer is similar to answer choice A. We need to address the defective enzyme, not prevent the enzyme from being expressed. Let’s keep comparing.
    3. No, because mRNA does not persist in the cytoplasm of the cell. This answer starts our better because it’s consistent with our “NO” prediction. Let’s see the reasoning. Persistence of mRNA means it continues to exist in the cytoplasm of the cell. The mRNA not persisting does not address the question properly. We’re not worried about the stability of our mRNA. The first part of this answer choice is better than both A and B. C is our best answer right now.
    4. No, because blockage of phenylalanine hydroxylase gene expression will not remedy the original disorder. This answer choice is consistent with what I said during the breakdown, and as I’ve gone through the past few questions. We have to know the cause of the malady, and treat that cause. The ineffective enzyme isn’t treated by blocking phenylalanine hydroxylase gene expression. Answer choice D is the best answer.

30) We know the antisense nucleic acids are complementary to the sense mRNA. We want to write out our antisense molecule so it’s complementary and antiparallel to the sense molecule. We know nitrogenous base A pairs with U, and G pairs with C. 

Our new strand will be antiparallel, so let’s write it out 3’ to 5’. We have G, C, U, A, U, G as our sequence. If we want to write it out from the customary 5’ to 3’, that would be G, U, A, U, C, G

    1. Answer choice A has T, which automatically lets us eliminate that answer choice
    2. Answer choice B has the sequence as our prediction, but backwards, or 3’ to 5’. 
    3. Answer choice C matches the sequence in our prediction. Note we have the same sequence as answer choice B, but it’s properly written from 3’ to 5’ in this answer choice. 
    4. Answer choice D incorrectly switches A with U and U with A. We’re left with our only answer that matches our breakdown and prediction, answer choice C.

31) To answer this question, we need to pick a way that involves delivering the antisense gene in a way that the gene can be incorporated into cells. The antisense gene needs to be present to ensure it can bind specifically to the sense mRNA and possibly prevent translation, as necessary. Typically, how this is done in gene therapy is by inserting or incorporating genes into a virus. That virus is then able to deliver the gene into a chromosome. Ultimately, we find an option that finds a way to incorporate the gene into the chromosome.

    1. orally as an emulsified product. This answer choice is unreasonable. The digestive system would break down this emulsified product and it would not be able to incorporate into the chromosome.
    2. microinjection into individual body cells. Theoretically, this strategy could work, until you think about the monumental task this is. The question stem we need an efficient method. This answer is effective, but doing a microinjection into every cell is far from efficient. Even though this answer choice isn’t the most efficient, it’s still more effective than answer choice A.
    3. intravenously as a nonantigenic, blood-stable product. This would get the gene into the bloodstream, but not necessarily into the cells themselves. This answer choice is efficient, but not as effective. It’s almost like the opposite of answer choice B, but it’s not as useful if it’s not effective. We’ll still stick with answer choice B as our best answer. 
    4. infection of an embryo by a virus modified to carry the gene. This answer choice mentions utilizing a virus modified to carry the gene. I mentioned we’re looking for an answer like this. Do we have to infect an embryo for this method to work? Not necessarily, but this ensures the cells that are derived from the embryo will contain the gene we want. Even though this isn’t the only way, this sounds like our best answer choice.

32) We’re presented a hypothetical situation here that normally doesn’t happen according to our passage. So if mRNA is not degraded by intracellular agents, then how would things differ? We’ll use the passage to see where the author talks about this degradation. It says “A problem encountered in the design of antisense drugs was that oligonucleotides may only persist for a matter of minutes before they are degraded by cellular processes.” So we’re essentially getting rid of this issue that we see in sentence 1. Now we’re saying these oligonucleotides don’t degrade in a matter of minutes. What would that mean? Well for starters, more mRNA, because it’s not being degraded. Also, there’s not the same turnover in mRNA. We’re not degrading at the same rate as before, and protein expression can change.

    1. The production of tRNA in the nucleus. Having more or less mRNA is not going to change the fact that tRNA is being produced. Theoretically more tRNA is being produced if there’s more mRNA, but the life span of the mRNA does not affect whether tRNA is produced or not. 
    2. The coordination of cell differentiation during development. This answer mentions something that would be affected by the turnover of mRNA. What do we know about cell differentiation? It requires the cells to become different, distinct proteins. So several mRNA are involved in this process. We need that mRNA turnover for the proper protein expression, not the same mRNA constantly. Compared to answer choice A, answer choice B is more affected by the change in time in degradation.
    3. The diffusion of respiratory gases across the cell membrane. This answer is out of scope. We’re not told this new surplus of mRNA is specific to this diffusion process across the cell membrane. There’s just no way to make this connection, and answer choice B and answer choice A are superior answers. We also established B is superior to A. 
    4. The replication of DNA in the nucleus. This answer is similar to option A. We’re expecting more mRNA, but the life span of the mRNA isn’t going to dictate whether there’s replication of DNA in the nucleus. This would still happen, even with the smaller lifespan. We can answer this question with answer choice B: the process that’s the most affected is the coordination of cell differentiation during development. 

Biology Question Pack Volume 1: Passage 6

33) We’ll reference the passage to see where the author mentions the breathing and heart rates, then we’ll use our general knowledge to explain the increase. It says “Although she was in excellent physical condition from daily swimming in the ocean, she noticed that the first time she went diving, she experienced an elevated pulse and ventilation rate.” In the passage we find out that Sarah is in excellent physical condition and she’s also used to swimming in the ocean daily. It’s likely Sarah shouldn’t have a very elevated pulse or ventilation during any non-strenuous activities. Her body also shouldn’t have any abnormal response to being in water.

The passage also says “By the third time she went diving, her heart and breathing rate were no longer elevated.” This is also strange because it was a very quick change from elevated heart rate and breathing rate to a normal level. If it was the actual activity causing the elevated rates, we should’ve seen a more gradual change back to normal levels, if at all. What’s the reasoning for this? It’s likely the first few times she was either excited or anxious, and that caused the elevated heart and breathing rate. 

    1. activation of the sympathetic autonomic nervous system by the new experience. This is consistent with our prediction. The sympathetic nervous system is the part of the autonomic nervous system. Under stress it raises blood pressure and heart rate, constricts blood vessels and dilates the pupils. That’s consistent with what we’re seeing, and our prediction. Let’s keep comparing.
    2. activation of the parasympathetic autonomic nervous system by the new experience. The parasympathetic nervous system tends to slow the heart and relax the muscles. So the opposite of what we’re seeing in Sarah. Answer choice B isn’t great.
    3. hypoxia caused by the inability of her blood hemoglobin concentration to supply sufficient oxygen for the strenuous exercise of swimming at sea level. Hypoxia happens when the body is deprived of oxygen, just like the answer suggests. But ultimately, we said during our prediction that Sarah swims in the ocean daily, so she shouldn’t have any negative effects from swimming at sea level. This answer choice is unreasonable-it doesn’t make sense given the circumstances. 
    4. elevated core body temperature caused by swimming in warm tropical waters. Sarah already swims in the ocean daily, and she was in the same temperature water just 3 days later without suffering the same effects. This is not a great option given this information. We stick with the best answer here: answer choice A.

34) Similar to our previous question, we want to explain the reason for elevated heart and breathing rates. But this time we’re focused on the snow skiing trip, and a prolonged increase in rates. The passage says “Again, she noticed that the first time she went skiing, her heart and ventilation rate were faster than usual. Although it was not as elevated by the end of the first week, her heart and breathing rates were still higher than usual.” So unlike when she went skin diving, Sarah’s heart and breathing rates remain somewhat elevated. We have to explain why that is. This could mean Sarah gets the same thrill every time she attempts skiing. It could also be that skiing is more strenuous than the activities she’s used to doing. She also may be having a tough time acclimating to the elevation in the mountains of Colorado. Considering that her heart and breathing rate dropped to normal after skin-diving a few times, we doubt it’s the thrill of the new experience after a full week. We find out later in the passage that there was not a significant different in the amount of work performed between skin diving and skiing, so it’s not that skiing is more strenuous. Our last prediction is that it could be the elevation in Colorado. The lower air pressure means taking additional breaths to get the same amount of oxygen.

    1. activation of the sympathetic autonomic nervous system by the new experience. This is the same answer we saw on our previous question. I said the sympathetic nervous system is the part of the autonomic nervous system. Under stress it raises blood pressure and heart rate, constricts blood vessels and dilates the pupils. This was possibly the case the first time Sarah went skiing, but not the cause of the prolonged increase. 
    2. activation of the parasympathetic autonomic nervous system by the new experience. The parasympathetic nervous system tends to slow the heart and relax the muscles. Another identical answer to our previous question. This is the opposite effect of what we see in Sarah’s body. We see elevated heart rate, so this contradicts the excerpt we got from our passage. 
    3. hypoxia caused by insufficient blood hemoglobin concentration to supply oxygen for exercise at the low oxygen pressure found at high altitudes. This answer choice matches our third reason and prediction. I said the elevation means lower air pressure and not enough oxygen to her body’s cells. Answer choice C takes the title of “best answer” from answer choice A. 
    4. depressed core body temperature (hypothermia) caused by exposure to cold temperatures at high altitudes. Hypothermia is the result of a dangerously low body temperature, not a slightly elevated heart rate. The experience would not be a pleasant one and Sarah would almost perpetually be shivering and feel extremely cold. This isn’t what we’re seeing. Answer choice C is the best answer choice.

35) We’ll need to think about the excretory system and osmoregulation, and how different enzymes regulate water levels in the body.

The excretory system helps regulate blood pressure and water levels. In this case, we’re seeing that heart and breathing rates are high, and there’s less water absorption. That typically means blood pressure is high. But let’s break down the question so we know how we come up with that conclusion. Usually in our science classes, we think about what happens with osmoregulation when blood pressure is low. So that’s what we’ll do here to start. The way we usually think about it is:

Blood volume is low: Renin stimulates the production of angiotensin I, which is converted into angiotensin II. Next, Aldosterone secretion from the adrenal cortex. That’s induced by angiotensin II. That causes the tubules of the kidneys to increase the reabsorption of sodium and water into the blood, and increases blood volume and blood pressure.

But the increased excitement, heart rate, and breathing rate in Sarah’s example, is the exact opposite. That means in Sarah we’re expecting the body to adjust by decreasing blood volume and blood pressure by decreasing reabsorption of sodium and water into the blood. The increased volume of urine corresponds to increased blood pressure and increased blood volume. 

    1. Answer choice A says increased blood pressure caused by her excitement or anxiety. This answer matches part of our prediction. I said the increased urine is most likely a result of the increased blood pressure. The body has to account for the increased blood pressure through osmoregulation, which includes increased urine. 
    2. Answer choice B says reduced blood pressure caused by her excitement or anxiety. This is the opposite of our prediction. The increased heart and breathing rate and increased urine means increased blood pressure. We just went through the reasoning in our question breakdown as well. 
    3. Answer choice C says absorption of water from the ocean. This is an unreasonable answer. Sarah is not absorbing water from the ocean. Our skin does not absorb water from water sources like the ocean or showers. Answer choice A is still our best option. 
    4. Answer choice D says inability to cool the skin through evaporative water loss. Even though there might not be the same evaporative water loss as we have during land exercise. This is not the reason for the increased amount of urine. The body is still sweating while doing strenuous swimming, even if there’s not the same evaporative water loss. We’ll stick with our best answer: Answer choice A. 

36) We’re asked which of the 3 injuries listed would be consistent with a level of myoglobin in the urine, and remember we can pick more than one option here. I know from going through the content outline, myoglobin is the oxygen binding molecule in muscle and organs. Myoglobin binds oxygen tighter than hemoglobin- which is the oxygen binding molecule in blood. If we have myoglobin in the urine, that means our injury is in muscles, or an organ. Of our choices, options 2 and 3 correspond to muscles and organs. We don’t find myoglobin in the bone.

That corresponds to answer choice D: II and III only. The physician detected myoglobin in the urine because of a damaged organ, like the kidney, or in damaged muscles. Answer choices A-C are all incorrect because of what we said during our breakdown. Always be careful with this format of question. Answer choice A includes “I. Broken bone” but we know that answer is incomplete. That’s the reason you should always look at every answer choice, even if it’s briefly.

37) This is going to come straight from our general knowledge. This could easily be a standalone or discrete question, but it’s loosely related to the topic covered in the passage, so it’s linked to the passage. Get your content down if you haven’t already so questions like this are automatic. 

-Involuntary respiration is controlled by the respiratory centers of the upper brainstem. Chemoreceptors will detect blood pH levels and adjust ventilation rate.

-The cerebellum governs balance and fine motor movements. Its main function is maintaining coordination throughout the body.

-The hypothalamus controls the basic drives for hunger, thirst, and sexual attraction. It also releases hormones and regulates emotional responses.

We’re looking for an answer that lists: brainstem, cerebellum, and hypothalamus.

We already broke down the question and the reasoning for our three choices. Our breakdown matches answer choice D. Answer choices A-C contradict our breakdown, and are incorrect.

38) This could also be another standalone question. We want to explain vasoconstriction and vasodilation, and why someone’s vessels might quickly dilate, even in cold temperatures.

The question stem mentions Sarah’s blood vessels were usually constricted in the cold temperature. That makes sense, it decreases blood flow to skin and helps minimize heat loss from warm blood and maintains a normal internal temperature. By why can’t they constantly be constricted? Why do her blood vessels occasionally dilate? The vessels constantly being constricted leads to superficial tissues to be deprived of the necessary oxygen and nutrients. The body has to adjust to this through the occasional dilation. Dilation is used to allow for greater than normal blood flow throughout the body. And providing these cells and the rest of the body with sufficient nutrients and oxygen.

    1. Maintain normal skin tone. This is not something that we expect to be maintained as closely by the cells. The question stem asks which is the most probable purpose. We’re focused on something that’s a higher priority in the cold temperature. That includes maintaining the body’s core temperature, and continuing to circulate and supply oxygen throughout the body.
    2. Maintain sufficient oxygenation of cells. This sounds like our prediction. In short periods of cold, the blood vessels being constricted is fine. But after long periods, the body hast to continue to supply the cells with more oxygen. This is a better option than answer choice A. We’re liking answer choice B.
    3. Reduce excessive blood pressure. The blood vessels in the skin are capillaries. Excessive blood pressure isn’t controlled through capillaries. Arterioles are the vessels that would control the blood pressure. Reduced blood pressure is a side effect of dilation. But there isn’t dilation because of the excessive blood pressure. 
    4. Maintain normal muscle tone. This is similar to answer choice A. When Sarah is in extremely cold temperatures, her body is focused on maintaining proper oxygenation and temperature. It’s not as focused on these secondary processes like maintaining skin tone and muscle tone. These are lower priority, so we’re sticking with our correct answer, answer choice B. 

Biology Question Pack Volume 1: Questions 39-43

39) To solve this problem, we want to point to the figure and show where neurons involved in feeling pain would theoretically be found. We have the painful stimulus shown as #1; that’s the actual stubbed toe. We have synapses at 2 and 3. Finally, we have the efferent nerve terminal at 4, which we can see is in the muscle. We’re looking for a combination of 2 different locations according to our possible answers. To finish, we’ll have to go over how reflex arcs work. 

When we have a stimulus like a stubbed toe, or a hot object, for example, there’s a reflex. The reflex arc is the path taken by the nerve impulses from the stimulus, to sensory neuron, to motor neuron, to reflex muscle movement.

Looking at our figure, we want to know where we’d find neurons that involve feeling pain. For this pain to actually be recognized, we’d need a pathway traveling up the CNS. Location 1, like the author shows, is where the stimulus is. That happens from the toe hitting an object. But we need a way to get to the CNS. That would come from neurons at positions 2 and 3. We need to add neurons at these points that would get that signal to the CNS, and ultimately the brain, where we actually perceive pain.

Position 4 is the neuromuscular junction. It wouldn’t make sense to add additional neurons here that go to the CNS.

Answer choice A says additional neurons could be placed at II and III which matches our breakdown. Quick side note: we were confident about position 4 not being a possible answer. That means we have another bit of information validating our answer. Answer choices B-D all contain option 4 so we were able to eliminate all of those answer choices and give us more confidence in answer choice A as our best answer.

40) To answer this question, we’ll go over what constitutes a eukaryote, and specifically which characteristics mark fungi as eukaryotes. 

What are the defining characteristics of eukaryotic cells?

1) A membrane-bound nucleus, a central cavity surrounded by membrane that houses the cell’s genetic material.

2) A number of membrane-bound organelles, compartments with specialized functions that float in the cytosol.

3) Mitotic division which results in two daughter cells each having the same number and kind of chromosomes as the parent cell.

That means we want something along the lines of: fungi have a membrane bound nucleus and organelles, and undergo mitotic division.

    1. They have cell walls. Not one of the criteria we laid out in our breakdown. In fact, most prokaryotic cells will have a rigid cell wall surrounding the plasma membrane. Plants have cell walls but vertebrates will not. This is not a great answer.
    2. They contain ribosomes. Ribosomes are found in both eukaryotes and prokaryotes. We have different subunits depending on if the ribosomes are in eukaryotes or prokaryotes, but the ribosomes themselves are present in both. Both answer choice A and B aren’t great. If we had to choose though, we should eliminate answer choice A, because cell walls are more associated with prokaryotes. 
    3. They contain mitochondria. This answer choice is consistent with our prediction. One of our criteria for eukaryotic cells is they have membrane-bound organelles, like mitochondria. This is a good answer choice for now, especially because answer choices A and B both contradicted our breakdown. Answer choice C is now the best option
    4. They exhibit sexual reproduction. This answer choice might seem attractive because we associate sexual reproduction more with eukaryotes. But remember, we’re dealing with fungi. Fungi will reproduce asexually, not through sexual reproduction. This goes against what was asked in the question and contradicts our breakdown, so we can eliminate answer choice D. We’re left with our correct answer, answer choice C: They contain mitochondria

41) Remember we’re focused on mammals. Mitosis and meiosis are both forms of division of the nucleus in eukaryotic cells. Mitosis and meiosis share some similarities but also significant differences, and that’s where we’re focused. Biggest difference in mammals is that mitosis produces genetically identical diploid daughter cells, and meiosis produces genetically different haploid cells. 

    1. Synapsis. This is the opposite of our breakdown. Synapsis is the pairing of homologous chromosomes, and binding together. This happens in meiosis I, but not in mitosis. This is the opposite of what we know from our general knowledge. 
    2. The splitting of centromeres. This answer choice ties into answer choice A. In meiosis I, we have the splitting of those homologous chromosomes we talked about in answer choice A. In mitosis, during anaphase, we have actual separation of centromeres toward opposite poles of the cell. Answer choice B is the best answer choice so far.
    3. The pairing of homologous chromosomes. This is another answer choice similar to answer choice A. We have pairing of homologous chromosomes in meiosis I. We talked about that when we discussed synapsis in answer choice A. We can eliminate answer choice C, and answer B is still our best option.
    4. The breaking down of the nuclear membrane. Nuclear membrane, just like it sounds, is the membrane around the nucleus. During prophase I, we have synapsis, and by the end of prophase I, the nuclear membrane will break down. Same as in mitosis where we also have the breakdown of the nuclear membrane. That means we can eliminate answer choice D. We’re left with our correct answer, answer choice B: The splitting of centromeres

42) To answer this question, we want to find one of the answer choices that is not a function of mammalian skin. That means 3 will be functions of mammalian skin, and 1 will not.

Biggest functions of the skin are protection, osmoregulation, and maintaining homeostasis. 

The skin provides an overlaying protective barrier from the environment and pathogens. It also contributes to the adaptive immune system. Along the same lines of protection, the skin helps protect our body’s internal structures from physical, chemical, biological, radiological, and thermal damage. We’re going to focus on protection, osmoregulation, and maintaining homeostasis. 

    1. sensation. The most superficial layer of the skin is the epidermis and functions include touch and temperature sensation. But we have to be careful here! We want an answer choice that is not a function of mammalian skin. That means answer choice A is not a great option.
    2. respiration. This answer choice would be a function of some organisms like insects, fish, amphibians, but not so much in mammals. Once again, we were looking for an answer choice that’s not a function of mammalian skin. This is a good answer choice and better than answer choice A.
    3. protection from disease. Protection, and specifically protection from disease was one of the main functions of mammalian disease. That means we can eliminate this answer choice because it directly contradicts our breakdown.
    4. protection against internal injury. Another answer choice that contradicts our breakdown. We mentioned skin helps protect the body’s internal structures from different types of damage. That means we can also eliminate answer choice D. We’re left with our correct answer, answer choice B: Respiration. 

43) To answer this question we’ll go through what we know about inbreeding, and the fitness of a population. Inbreeding is the mating of closely related individuals. That can have an undesirable effect of bringing together harmful recessive mutations. These mutations can cause abnormalities and susceptibility to disease. Inbreeding increases the frequency of homozygotes, decreases heterozygotes, and decreases genetic diversity in general.

    1. genetic diversity of the population. This is the opposite of our breakdown. Inbreeding decreases genetic diversity because there’s mating of closely related individuals. This goes against what the question is asking. We’re looking for something that increases, so we’ll keep comparing and we don’t like answer choice A for starters.
    2. levels of aggression in the population. This answer choice is not correlated to inbreeding. We went through the effects of inbreeding on a population, but as far as the MCAT is concerned, the levels of aggression are not going to change because of this inbreeding. That being said, answer choice A was a direct contradiction, so answer choice B is now our superior answer choice.
    3. rate of spontaneous mutations. Spontaneous mutations come more from errors in DNA replication, lesions, or transposable genetic elements. None of these are the result of inbreeding. Spontaneous, by definition, means there isn’t really an external stimulus. This is another neutral answer choice like answer choice B, but neither one jumps out as a clear #1 choice just yet.
    4. incidence of expression of deleterious recessive traits. This answer choice is interesting. Deleterious genes are harmful, so we don’t see these genes as much because of natural selection. Luckily, individuals carrying a recessive deleterious allele will still be healthy. But remember, in inbreeding there’s mating of closely related individuals. That means in inbreeding, there’s a decent chance an individual with a recessive allele will mate with another individual that also has that recessive allele. 

Even if two individuals have a single recessive allele, there is that chance that their offspring will express the recessive traits-just due to probability. That makes this a superior answer choice to answer choices B and C, which were neutral, but didn’t directly answer the question being asked. We’re left with our best answer, answer choice D: incidence of expression of deleterious recessive traits. 

Biology Question Pack Volume 1: Passage 7

44) The passage says “The Tm is the maximum rate of transport (mg/min) at which a substance can be reabsorbed by the kidney.” Immediately after it says “That is, if the filtration rate of a substance exceeds its Tm, the substance will begin to appear in the urine.”

Combining those two sentences, we can say that once this maximum rate of transport is reached, we’ve essentially saturated the kidneys with the substance, so there can’t be additional absorption. And what happens when a substance isn’t reabsorbed? The substance will instead be excreted from the body. We’ll start seeing the substance in the urine. 

    1. Answer choice A says rate of concentration of the substance in the glomerular filtrate. The Tm is the maximum rate of transport at which a substance can be reabsorbed by the kidney. That means we’ve saturated the kidneys, and additional substance can’t be absorbed. I’d say this answer choice is out of scope. Exceeding the rate of concentration of the substance in the glomerular filtrate is not relevant to the answer we’re looking for.
    2. Answer choice B says rate of concentration of the substance in the urine. I’d contend this answer choice is also out of scope. Tm represents the point at which the kidneys are essentially saturated. At that point any additional substance is excreted. The rate of concentration of the substance in the urine itself does not specifically have to do with Tm. I can’t really decide between A and B just yet. We’ll keep looking at the remaining answers and see if one sticks out and we can eliminate both A and B for being out of scope.
    3. Answer choice C says capacity of the kidney tubules to reabsorb the substance. This is very similar to what we covered in our breakdown. Tm does represent the rate of filtration that just exceeds the capacity of the kidney tubules to reabsorb the substance. That means additional substance can’t be absorbed, so it’s excreted from the body instead. Neither answer choice A nor B answered the specific question being asked like answer choice C. I like answer choice C right now.
    4. Answer choice D says capacity of the bladder to store and excrete the substance. This answer is out of scope. The author doesn’t mention the bladder’s ability to store the substance in the passage, and we don’t mention it in our breakdown of the question. Best answer here is going to be answer choice C.

45) We’re looking for a quantitative value here: the amount of glucose we’re expecting in the urine as a rate, given normal conditions. We can reference the passage for the exact value of Tm for glucose in a normal human. We’ll compare that value to this tubular load of approximately 125 mg/min.

The passage says “The Tm for glucose averages 320 mg/min in an adult human.” 

We’re given the average tubular transport maximum for glucose here. Having too much glucose in the urine is usually the sign of health issues. We’re expecting this 320 mg/min to be much higher than the normal filtered amount of glucose we typically see. That means if we have a smaller tubular load of glucose, like in our question stem, it’s all likely to just be reabsorbed. That means we’re expecting 0 mg/min glucose in the urine under normal conditions.

We came up with an exact value, and none of our answer choices here are very close to 0 mg/min except answer choice A. In fact, they are all much higher than what we would expect.  

46) We can answer this question using our general knowledge, and how blood pressure relates to clearance and the formation of urine. We won’t need to rely much on the passage. Glomerular filtration rate (or GFR) is the volume of glomerular filtrate formed per minute by the kidneys. A higher GFR means the kidneys are filtering blood faster. Naturally, that rate increases due to high blood pressure. When blood pressure is lower, there’s more reabsorption. Why is that? Blood is being filtered more slowly, allowing for more reabsorption of solutes like substance A. Think back to the passage, there’s a tubular transport maximum of a substance that affects plasma clearance. That Tm is the maximum rate of transport at which a substance can be reabsorbed by the kidney. When the GFR is lower, there’s more reabsorption possible.

    1. An increase, because the concentration of Substance A in the urine will increase. This is the opposite of our prediction. We expect clearance to decrease. The lower blood pressure means there’s more time for reabsorption of solutes like substance A in the kidneys. We’re expecting a lesser concentration of substance A in the urine. 
    2. Answer choice B says An increase, because the ADH levels will be very low. This is similar to our last answer choice because we’re expecting clearance to decrease. Low ADH levels can correlate to lower blood pressure, but low ADH levels will not cause plasma clearance to increase. We would have the opposite effect. Low ADH levels that lead to lower blood pressure would mean more reabsorption. 
    3. Answer choice C says A decrease, because the decreased rate of urine output will allow more reabsorption by the kidney. This is consistent with our breakdown of the question. We’re expecting the rate of plasma clearance to decrease, and this reasoning is consistent with our prediction. Lower-than-normal blood pressure correlates to decreased rate of urine output. There’s more time for reabsorption of solute by the kidneys. The rate of plasma clearance of Substance A is less. This is the best answer choice of the 3 so far.
    4. Answer choice D says A decrease, because ADH levels will be very high. The first part of this answer choice is consistent with our breakdown. I am looking for an answer choice that mentions a decrease in the rate of plasma clearance. But ADH levels being very high corresponds to higher blood pressure. The author actually mentions this in the passage with the second dosage of ADH. Answer choice C is the best option here.

47) We can answer this question using the passage, and interpreting Figure 1.

I’ve got Experiment 1 and Figure 1 from the passage here. We’re told in the question stem that we have equal concentrations of Substance A and glucose in the plasma. So, let’s go along our x-axis, which is concentration in the plasma, and mark 8 mg/mL. What do we notice? We have a concentration of roughly 2 mg/mL in the urine of Substance A. We have no glucose concentration in the urine whatsoever. What does that tell us? The kidney will be clearing Substance A from the plasma more rapidly. We’re expecting glucose to be reabsorbed at this concentration.

    1. Substance A, because the slope of the clearance line for Substance A is higher than that for glucose. First part of this answer choice is consistent with our prediction, we expect solution A to clear more rapidly. But then we get to the reasoning. There are two things wrong with this reasoning. The X-intercept for substance A is found at a lower plasma concentration, meaning clearance is happening at a lower concentration. And the slope of the line for the two substances is not relevant. 
    2. Substance A, because Substance A reaches its Tm at a lower plasma concentration than does glucose. This answer choice is consistent with our prediction. The X-intercept for substance A is found at a lower plasma concentration. What does that mean? Substance A has reached its Tm, and the kidneys can no longer absorb all of the substance A in the plasma. At that point we start seeing substance A in the urine. Answer choice B is the superior answer choice at this point. 
    3. Glucose, because glucose reaches its Tm at a higher plasma concentration than does Substance A. First part of our answer choice is inconsistent with our prediction, even if the reasoning is true. We always want to be careful to pick the best answer. Because glucose reaches its Tm at a higher plasma concentration, glucose is actually cleared more slowly. I am not a fan of answer choice C.
    4. Glucose, because the slope of the clearance line for glucose is lower than that for Substance A. Another contradiction. We expect substance A to clear more quickly. The slope of the clearance line is not relevant here. Rather we’re more concerned with the X-intercept and the Tm of the two substances. Answer choice B remains the best answer choice here.  

48) I want to answer this question using the passage, and interpreting Figure 1. I’ll find the approximate plasma concentration of glucose where Tm is reached. Note, the answers are given in units of mg/mL.

See above: we have Experiment 1 and Figure 1 from the passage here. We want to find Tm, which, from the passage, is the maximum rate of transport (mg/min) at which a substance can be reabsorbed by the kidney. We also know, if the filtration rate of glucose exceeds its Tm, then glucose will begin to appear in the urine. In other words, we want to find the plasma concentration at which glucose first begins to appear in the urine. This is as simple as looking at our graph and finding the X-intercept. At a plasma concentration of 10 mg/mL, we begin to see there is glucose in the urine. 

We solved for an exact value (10 mg/mL). None of our answer choices are especially close to one another, and the units are all the same. Pick the correct answer, answer choice B: a plasma concentration of 10 mg/mL. Make sure to not pick answer choice A, which incorrectly shows the X-intercept of the Substance A line instead of the glucose line.

49) I’ll attack this question by using the passage, and interpreting Experiment 2. I’ll also have to know the relationship between blood pressure and urinary output. Let’s go back to Experiment 2:

This excerpt shows the details of Experiment 2. We’re told “The second dosage, which was much higher than the normal physiological concentration, affected urine output by increasing blood pressure.” We’re not focused on the ADH itself, but rather those last 7 words in this paragraph. How the increased blood pressure affects urinary output. 

We’re going to revisit something we covered in question 46. Glomerular filtration rate (or GFR) is the volume of glomerular filtrate formed per minute by the kidneys. A higher GFR means the kidneys are filtering blood faster. Naturally, high blood pressure causes an increase in the flow of fluid through the kidney’s system. That will also mean more urinary output because there’s less absorption.

On the other hand, when blood pressure is lower, there’s more reabsorption. Why is that? Blood is being filtered more slowly, allowing for more reabsorption. 

Ultimately, the higher blood pressure increased the GFR and that decreased the absorption by the kidneys.

    1. glomerular filtration rate. This is consistent with what I just said in my breakdown of the question. The higher blood pressure in the body causes an increase in the flow of fluid through the kidney’s system. This increased filtration rate means less time to absorb water, and ultimately more water leaves the body through urine.
    2. Tm of solutes. The increased blood pressure does not increase this Tm value. Tm is the maximum rate of transport at which a substance can be reabsorbed by the kidney. It doesn’t vary with changes in blood pressure like answer choice B is insinuating. Answer choice A is the superior answer so far.
    3. water reabsorption from the tubules. This is the opposite of our prediction. We said the increased blood pressure corresponds to increased urinary output. By increasing water reabsorption, that decreases the volume of urine.
    4. concentrating ability of the loop of Henle. This is similar to answer choice C. Concentrating of the loop of Henle also means additional water absorption. Note both C and D are implying there is additional water reabsorption, and less volume of urine. We can’t have two correct answers for a question, and sometimes that’s a way to eliminate answer choices: if they’re essentially saying the same thing. Once we eliminate answer choices C and D, we’re left with our correct answer: answer choice A.

Biology Question Pack Volume 1: Passage 8

50) We need to reference and interpret Figure 1 from the passage (I tried my best to ensure the writing here is a proper size. It may help to have the passage pulled up separately if the numbers are hard to see). We ultimately want to show that the effects of ACH are seen when endothelium is intact.

Above I have Figure 1 from the passage. We need to explain how we can conclude that intact endothelium is necessary for the relaxation of smooth muscle when ACH is applied. Top graph shows our aorta with endothelium. Bottom graph shows aorta without endothelium. There’s norepinephrine added to both on the left side of these graphs that causes constriction. Theoretically, this tension should go away with the addition of ACH. 

We see that when an ACH concentration of 10^-7 M lowers the tension in the aorta with endothelium. But if we look at the bottom graph, there’s no concentration of ACH that causes tension to decrease. Only the sample with endothelium is responsive to ACH, and we can see that right as the ACH with concentration 10^-7 M is added to the sample.

    1. The tension initially changes when norepinephrine (NE) is added. This answer is technically true, but it’s not answering the specific question being asked. We want to know about the relaxation of the smooth muscle, and the response to ACH, not norepinephrine. Norepinephrine causes constriction, not relaxation. 
    2. The tension decrease occurs more quickly in the ring without endothelium. This answer contradicts our figure. We mentioned there is no tension decrease in the ring without endothelium when ACH is added. This answer choice is testing whether we look through the figure carefully. 
    3. The tension decreases upon addition of 10-7 M ACH, only in the ring with endothelium. This is consistent with our prediction. We said the tension of the sample with endothelium decreases once the 10^-7 Molar ACH is added. Even when ACH of a higher concentration is added to the aorta without endothelium, there’s no decrease in tension. This answer choice supports the conclusion given in the question stem. This is better than answer choices A and B.
    4. The tension decreases during washout in the ring without endothelium. This is another answer that’s technically true. During washout the norepinephrine and ACH are removed, and that lowers the tension in the ring without endothelium. But this does not tie into the conclusion that ACH is only effective when there is intact endothelium. We want to focus on the response to different concentrations of ACH applied to our samples, not the washout. This doesn’t answer the specific question being asked, so we’re left with our best answer, answer choice C.

51) Once again, We’ll need to reference and interpret Figure 1 from the passage. We ultimately want to show the sensitivity of the aortic smooth muscle to ACH. This question is a bit open-ended, so we can quickly glance at our answer choices. Normally I don’t like doing this before breaking down the question because then we approach the question with biases. But in this case, we need to narrow down our thinking a bit to know what the test maker is asking. It looks like we want to know how the presence of norepinephrine and the presence of the endothelium affect ACH sensitivity.

First thing we want to note is there is nothing conclusive we can determine from the presence of norepinephrine. Adding norepinephrine causes tension in the two rings, but that tension was necessary to test the effect of ACH. We’re not focused on the combination of ACH and norepinephrine. In terms of sensitivity of the aortic smooth muscle to ACH: ACH concentration of 10^-7M is what caused the tension to decrease significantly in the ring with endothelium. As far as the ring without endothelium, there’s no response to any of the shown concentrations of ACH.

    1. decreased by the presence of norepinephrine. I previous mentioned the presence of norepinephrine was part of the experiment, but there is nothing conclusive we can determine from its presence. The norepinephrine was present in both samples with the intention of increasing tension. That was its only use. We can’t conclude anything the sensitivity of aortic smooth muscle to ACH while comparing with norepinephrine.
    2. increased by the presence of norepinephrine. This is similar to answer choice A. And something else we want to note: if the presence of norepinephrine was the main driving force of the sensitivity of aortic smooth muscle to ACH, then we would theoretically pick either answer choice A or B. Unfortunately, the data does not show us that there is increased sensitivity in the presence of norepinephrine. Neither A nor B are great so far, so we’re looking for a better answer choice with C and/or D.
    3. increased at least 10 times by the presence of the endothelium. This answer choice is implying that the highest sensitivity of aortic smooth muscle with endothelium to acetylcholine is 10^ -7 Molar, like we said. And this sensitivity is at least 10 times greater than in the ring without endothelium. How would we test that? We find a concentration that’s a 10x factor greater or lesser than 10^-7 Molar and note the response of the ring without endothelium. At the concentration of 10^-6 M, there was no response to ACH. That means the presence of endothelium does increase sensitivity by at least 10 times. Answer choice C is the best answer so far. 
    4. greatest at 10-8 M, with or without endothelium. This answer contradicts the passage. We said in terms of sensitivity, ACH concentration of 10^-7 Molar is the greatest change in the sample with endothelium. There is practically no change in either sample when ACH concentration of 10^-8 Molar is added. Answer choice D is inconsistent with Figure 1 and the passage. Answer choice C is our best answer here. 

52) We need to reference Figure 1 once again, and this is actually something we’ve touched on while going through our previous questions. 

One more time we have Figure 1 from the passage. We want to point out the range where muscle tension is most sensitive to ACH. We’ve discussed this at length already. The ring without endothelium isn’t sensitive to ACH in the concentrations given in our figure. The ring with endothelium is most sensitive to ACH at a concentration of 10^-7 Molar. Notice the steep slope after the 10^-7 Molar ACH is added. We want our answer to be closest to that number.

    1. less than 10-8 M. Tension actually increased after the addition of this ACH.
    2. near 10-7 M. This matches our initial prediction, but let’s keep going through the other answers to be thorough.
    3. greater than 10-6 M. Tension was already decreased significantly when this ACH was added. We’re sticking with answer choice B.
    4. much wider in the ring without endothelium than in the ring with endothelium. The ring without endothelium was not sensitive to ACH at all. Correct answer here is still answer choice B.

53) This could easily be a standalone or discrete question even. It’s only slightly related to the topics in the passage, so it’s included here.

Blood pressure is the pressure of the blood against the walls of blood vessels. What determines this value? We have heart rate and stroke volume that determine cardiac output. We also have the tension in the blood vessels that can either reduce or increase blood flow. Vasoconstriction can reduce blood flow and increase blood pressure, for example. 

    1. the blood concentration of L-NMMA (which was our inhibitor) and norepinephrine. This is a very narrow answer, but these two factors do affect one of the main factors we mentioned. Both can affect blood flow. The presence of both will typically lead to vasoconstriction, and ultimately an increase in blood pressure.
    2. the cardiac output and the resistance to blood flow. This sounds like a better, more neutral, and more broad answer. What do I mean by that? Answer choice A only provided two ways by which blood pressure can be affected. Answer choice B encompasses both by including “the resistance to blood flow.” In addition to the resistance, we also have cardiac output, which considers heart rate and stroke volume. Even though answer choice A is listing two factors that can affect blood pressure, this is a much more complete answer. 
    3. the blood volume and the amount of L-arginine in the diet. This is another incomplete answer. Blood volume is the volume of blood in the circulatory system in an individual. Technically this does affect blood pressure, but it’s not a main determinant. L-arginine is similar to the options in answer choice A. There’s no mention in the passage, or in our prediction of L-arginine in the diet determining blood pressure. Answer choice B is still the superior answer choice. 
    4. the heart rate (heartbeats/minute) and the cardiac stroke volume. This answer gets us to cardiac output, which is half of our complete answer. But it doesn’t consider the resistance to blood flow. Even though a few of our other answers are partially correct, answer choice B is the best, and most complete answer. That means our answer is answer choice B: the cardiac output and the resistance to blood flow.

54) This is another question that could easily be a standalone question. We’re going to have to know about vasoconstriction obviously, and when changes in blood flow are necessary. We know vasoconstriction affects blood flow, and as a result, blood pressure. Generalized vasoconstriction usually results in an increase in systemic blood pressure. But vasoconstriction may also occur in specific tissues, causing a localized reduction in blood flow. That means we want a situation in which we have a decrease in blood flow, or an increase in blood pressure.

    1. Causing the decrease in blood pressure associated with fainting. This answer choice contradicts our prediction, and our general knowledge. Vasoconstriction causes an increase in blood pressure by narrowing vessels.
    2. Increasing blood flow to muscle during exercise. This answer also contradicts our prediction and general knowledge. Vasoconstriction narrows vessels, causing less blood flow to muscles. This would be a result of vasodilation.
    3. Increasing blood flow to skin during blushing. Similar to our last answer, this answer also contradicts our prediction and general knowledge. Vasoconstriction narrows vessels, causing less blood flow to muscles. This would also be a result of vasodilation. 
    4. Maintaining blood pressure during a hemorrhage. This answer choice is consistent with our prediction. During our prediction, we said vasoconstriction can cause a localized reduction in blood flow, and that’s exactly what has to happen here. A hemorrhage can mean a major loss of blood and severe consequences if not addressed quickly. Vasoconstriction can reduce blood flow to the area, minimizing blood loss. And work on maintaining blood pressure in the body, despite the decrease in blood. Answer choice D is the only answer that’s consistent with what we’re looking for in the question stem. 

55) We can go back to our passage and see what the author mentions about the competitive inhibitor and nitric oxide.

Above I’ve pulled an excerpt from our passage, it tells us that L-NMMA is a competitive inhibitor of the enzyme responsible for nitric oxide synthesis. If we inhibit this enzyme, then eventually we have no more nitric acid being made. What’s the function of nitric acid? It’s a relaxing substance and it causes adjacent arterial smooth muscle to relax. That means we can predict that the previously relaxing aortic ring will increase its tension, and not be relaxing long term.

    1. increase its sensitivity to acetylcholine. This answer choice suggests that adding the competitive inhibitor will affect the sensitivity of the relaxed aortic ring to acetylcholine. According to the passage, that sensitivity is the result of having an intact endothelium. According to what we’re given in the passage, adding the competitive inhibitor will affect NO synthase and nitric oxide production only. 
    2. cause the ring to dilate (reduce its tension). This is the opposite of our prediction. We said that adding the competitive inhibitor will decrease the relaxing effect. We’re expecting an increase in tension. This answer explicitly goes against our breakdown and the passage.
    3. cause the ring to contract (increase its tension). This answer choice matches our prediction. If we inhibit NO synthase, the relaxing aortic ring would increase its tension. There’s no more nitric oxide causing that relaxing effect. Answer choice C is superior to answer choices A and B.
    4. prevent norepinephrine from increasing ring tension. This answer is similar to answer choice A, because we can’t make this connection given the information we have in the passage. There’s no mention of this inhibitor also preventing norepinephrine from increasing ring tension. We know that increase happened in both of our samples in our experiment, so we expect it to continue to happen here. That means we can eliminate answer choice D and we’re left with our correct answer, answer choice C: cause the ring to contract (increase its tension).

Biology Question Pack Volume 1: Passage 9

56) First thing I want to note here is we’re seeing a reduction of something. So an answer choice will be reduced following the uncoupling of oxidative metabolism from ATP. I’ll use some details from the passage, but most of this question is going to involve using our general knowledge.

We know that when vertebrates ingest DDT, that DDT can accumulate in fatty deposits of the testis. The question stem says this can cause reduced fertility, and gives us a reason for this reduced fertility. It’s because of the uncoupling of oxidative metabolism from ATP production. Now, all we have to do is relate this uncoupling and decreased ATP production. Cellular respiration is a set of processes that convert energy from oxygen and nutrients to ATP. DDT uncouples the processes so that no ATP is produced: ATP synthesis in the mitochondria is inhibited. Without enough ATP, cells can’t carry out the reactions they need to function. So how do we relate that to the male reproductive system? Sperm cells are differentiated into the head, neck, middle piece, and tail. The middle piece of the sperm contains many mitochondria. And why is that? To generate ATP for motility. So that’s going to be our prediction for answering this question. Like I mentioned, we can use the information from the passage, but this question ultimately boils down to reasoning out how our content outline relates to what we read in the passage.

    1. glucose concentration of semen. In both the passage and the breakdown of the question, we didn’t mention anything about glucose concentration of semen, or anything beyond ATP production. We said the middle piece of the sperm contains mitochondria. And that’s to generate ATP for motility. That’s the answer we’re looking for, not glucose concentration of semen.
    2. testosterone concentration of semen. This is the responsibility of luteinizing hormone (LH). LH enters the testes to make and release testosterone into the testes and blood. Again, not something we saw in the passage or in our breakdown of the question. That’s a good indicator that we’re looking for a better answer choice here.
    3. blood circulation in the testes. We’re 0/3 here. None of the answers match our prediction, but none of them contradict our prediction either. A lot of the times we’ll have answers that do that. Let’s go through our last answer choice here, and then we’ll choose using what we know about ATP and the male reproductive system. 
    4. sperm motility. Looks like our patience paid off here. We found an answer that matched our breakdown, and was most relevant to the question stem. This answer is superior to answer choices A-C. We already broke down the relationship between ATP and motility.

57) We can go back to the passage, then use our general knowledge to finally decide the correct answer.

Above I’ve added paragraph 3 from the passage. We’re told burning and itching occur when dissolved DDT is applied to skin. We’re told sufficient levels of exposure can lead to pain. And ultimately, we’re given a hypothesis that tries to explain why these symptoms occur. That sentence is the key to answering our question. We asked the cause of the unpleasant symptoms, and this sentence says, “According to one hypothesis, these symptoms occur because DDT becomes incorporated into nerve cells, allowing Na+ to diffuse freely through axonal membranes.” We know from our content review that after resting potential is established, cells can be depolarized. What are we seeing here? These sodium ions can diffuse freely, and that’s what happens during depolarization. DDT is affecting neural impulses, or action potentials. And we’re seeing these neurons depolarized.

    1. motor neurons are depolarized. We said the diffusion of sodium cations happens during depolarization. What are motor neurons? Motor neurons transmit information from the brain to effector cells in the body. Is that what’s happening here? We’re likely dealing with a different type of neuron considering the sensations listed in the question stem. Sensory neurons convey information from tissues and organs into the central nervous system. So essentially converting external stimuli into electrical impulses. Let’s see if we can get a better answer choice.
    2. motor neurons are hyperpolarized. We’re not happy with the first part of this answer choice because it mentions motor neurons and we want an answer choice that talks about sensory neurons. What happens during hyperpolarization? So after depolarization, voltage-gated sodium ion channels are closed, and there’s movement of potassium. This is repolarization. Following repolarization, there is an overshoot in the potential of the cell because of this movement of potassium. We have an even greater negative charge than in the resting state. Answer choice A is still superior because it mentions depolarization.
    3. sensory neurons are depolarized. So again, we have depolarization, which we liked in the earlier answer choice. Only now we have sensory neuron instead of motor neurons. We said Sensory neurons convey information from tissues and organs into the central nervous system. That’s exactly what we see here. This is a superior answer to answer choice A.
    4. sensory neurons are hyperpolarized. First part of this answer choice is solid, but we have another answer choice that mentions hyperpolarized instead of depolarized. That ultimately contradicts what I said during the breakdown of the question and the reasoning I used to eliminate answer choice B. Once we eliminate answer D as well, we’re left with our correct answer, answer choice C: sensory neurons are depolarized

58) Be careful with the verbiage here. We’re going to find an answer choice that doesn’t reflect a function of the liver. It says “all of the following will be impaired EXCEPT.” The liver plays a vital role in the digestion of fats, and detoxifying the blood. It produces bile that’s required for the breakdown of fatty components of the food in the duodenum. The liver is mostly composed of hepatocytes which are involved in the synthesis of cholesterol, bile salts, and phospholipids as well. When there is excess glucose in the blood, it’s stored in the liver as glycogen.

    1. absorption of fats in the small intestine. We said the liver produces bile while helps in the digestion of fats, and absorption in the duodenum. This goes against what we’re looking for in this question. Make sure you remember the “EXCEPT” in the question stem.
    2. production of bile. We actually just mentioned this function in our previous answer choice, and in our prediction. The liver produces bile to break down fats. Again, this directly contradicts our breakdown of liver functions because this is a function of the liver. We’re looking for a function that’s not related to the liver. 
    3. detoxification of poisons. This is another answer we mentioned in our prediction. We said the liver detoxifies the blood-it removes ammonia from blood. It also metabolizes alcohol, drugs, and toxins. Another answer that doesn’t fit our criteria and similar to answer choices A and B.
    4. regulation of blood pressure. This is the only answer choice that wasn’t explicitly mentioned in my breakdown of liver functions. Regulation of blood pressure isn’t a significant function of the liver, so accumulation of DDT in the liver would not significantly affect blood pressure regulation. That means we’re left with our correct answer, answer choice D: regulation of blood pressure.

59) The passage mentioned DDT disrupts normal cell function, division, and growth. But ultimately, the answer is going to come from our general knowledge. We’re going to have to decide which structure was most likely damaged to initiate cancer or cause a mutation. Mutations are changes that occur in DNA sequence. They can disrupt regular gene activity and cause diseases, like cancer. Cancer is caused by mutations occurring in several growth-controlling genes. So essentially, we’re looking at an answer that mentions an error during DNA replication, damage to DNA, or issues with genes.

    1. Nuclear envelope. Nuclear envelope surrounds the nucleus of a cell. The nucleus contains almost all of the cell’s genome, so this answer is certainly in play. Damage to the nuclear envelope could expose the nucleus, and put DNA in harm’s way.
    2. Chromosome. Chromosomes are structures of nucleic acids and protein found in the nucleus of most living cells. Chromosomes carry genetic information in the form of genes. I said in our breakdown, if there are issues such as mutations in genes, this can lead to cancer. Even though answer choice A can lead to mutations, damage to chromosomes would directly cause a mutation or initiate cancer. This answer choice sounds like a frontrunner for now.
    3. Ribosome. Ribosomes are macromolecules made up of rRNA and proteins. They act as the sites for protein synthesis. Even though protein synthesis is adversely affected, we’re not seeing changes in DNA sequence, or any disruption of gene activity. 
    4. Histone. Histones are water-soluble proteins that are rich in the basic amino acids lysine and arginine. Histones are complexed with DNA in the nucleosomes of eukaryotic chromatin. Histones are found in the nucleus, but we said mutations are going to occur from damage to the DNA itself. Best answer here remains answer choice B: chromosome. 

Biology Question Pack Volume 1: Questions 60-64

60) In other words, what happens to pepsin when pH falls below 1.0? We have to think about what happens to enzymes, and proteins in general, under extreme conditions. We know from our content a protein’s function depends on the interactions within its folded structure. If the protein is subject to changes in temperature or pH, or exposed to certain chemicals, the internal interactions between the protein’s amino acids can be altered. That alteration may change the 3D shape/folding of the protein (tertiary structure). Although the amino acid sequence (also known as the protein’s primary structure) does not change, the protein’s shape or the way it is folded may change to an extent that it becomes dysfunctional, in which case the protein is considered denatured. That’s what we fear might happen to pepsin. At the optimum pH it has a specific 3D structure, but it can get disrupted at a pH below 1.0. 

    1. pepsin is feedback-inhibited. While this may or may not be true, we always have to consider every answer within the context of the question being asked. Feedback inhibition is not the reason pepsin catalysis is occurring more slowly at the very low pH below 1.0.
    2. pepsin synthesis is reduced. Pepsinogen and gastric lipase are secreted by chief cells. Acid converts inactive pepsinogen to pepsin which would still be taking place. Be careful with the verbiage here. We’re told catalysis occurs very slowly, not that there is a lack of pepsin in the stomach.
    3. the peptide bonds in pepsin are more stable. The stability of peptide bonds isn’t going to cause catalysis to occur more slowly. In fact, more stable bonds would mean the 3D shape/folding is less altered than we would expect. This answer choice is implying the opposite of what we’re looking for in our correct answer. Answer choices A and B are still superior to C because they don’t directly contradict what we’re expecting in the low pH situation. 
    4. the three-dimensional structure of pepsin is changed. This answer choice is consistent with my breakdown of the question. I mentioned extreme conditions like extreme temperature or pH can affect a protein’s function because the 3D shape/folding of the protein is altered. This answer choice is saying exactly this, so we’re going to stick with answer choice D as our best answer. 

61) To answer this question, we have to consider what happens when there is an increase in albumin levels. We can think about the function of albumin in the body. Albumin is a water-soluble family of proteins. In the liver, fats can be mobilized and transported via this blood protein albumin (or other lipoproteins). Albumin’s also the major osmoregulatory protein in the blood. Normally, the concentration of solute in the blood and tissue fluid is the same. However, increases in albumin in the blood lead to an increase in interstitial fluid into the bloodstream, and therefore an increase in blood pressure. Think how osmosis works. Water flows from an area of low solute concentration to an area of high solute concentration to get back to the same concentration of solute in the blood and tissue fluid.

    1. increase in the immune response. Think back to my breakdown and your content. Does albumin play a role in the immune response? It does not. We want an answer choice that is most likely to happen, and answer choice A is not a great option because an increase in the immune response is not most likely to happen.
    2. increase in tissue albumin levels. The albumin levels in the blood won’t affect tissue albumin levels. We see movement of water that affects blood volume, but not the movement of albumin itself.
    3. outflow of blood fluid to the tissues. We don’t expect fluid to flow out of the blood. Think back to our breakdown. We said increases in albumin in the blood lead to an increase in interstitial fluid into the bloodstream, and therefore an increase in blood pressure. This is the opposite of what we expect. Not a good answer choice. 
    4. influx of tissue fluid to the bloodstream. This is the opposite of answer choice C, which actually makes this a great option! Increases in albumin in the blood lead to an increase in interstitial fluid into the bloodstream. That exactly what this answer choice is saying. That makes answer choice D the correct answer.

62) To answer this question, we have to think about what happens following ingestion of excessive quantities of NaCl, and how the kidneys play a role. Kidneys regulate the osmotic pressure blood through filtration and purification, or osmoregulation. The kidneys also play a role in blood pressure control, using hormones that regulate blood pressure and water balance. The kidneys secrete renin directly into circulation that through a chain reaction results in the formation of angiotensin II, a potent vasoactive peptide that causes blood vessels to constrict, resulting in increased blood pressure. Angiotensin II stimulates the secretion of the hormone aldosterone from the adrenal cortex which is what we’re focused on for this question. Aldosterone causes the tubules of the kidneys to increase the reabsorption of sodium and water into the blood. This increases the volume of fluid in the body, which also increases blood pressure. If we have ingestion of excessive quantities of NaCl, we don’t have the same need to increase reabsorption of sodium, and we don’t expect an increase in the level of plasma aldosterone.

    1. No; aldosterone causes Na+ reabsorption by kidney tubules. We broke this question down thoroughly and this is exactly what we came up with. Aldosterone causes sodium reabsorption and decreased sodium levels in urine. Following ingestion of excessive quantities of NaCl, there’s no need for this reabsorption, or increased levels of plasma aldosterone. Let’s be thorough and check our other answer choices. 
    2. No; aldosterone causes Na+ secretion by kidney tubules. First part of this answer choice is consistent with our breakdown. We don’t expect an increase in the level of plasma aldosterone. However, aldosterone causes sodium reabsorption, not secretion by kidney tubules. Answer choice A remains superior.
    3. Yes; aldosterone causes Na+ reabsorption by kidney tubules. We don’t expect an increase in the level of plasma aldosterone. Despite the reasoning being correct where aldosterone causes sodium reabsorption, the first part of this answer is incorrect.
    4. Yes; aldosterone causes Na+ secretion by kidney tubules. Both parts of this answer choice are incorrect. We’ve gone through the reasoning here at length in our breakdown, and in answer choices A-C. We can stick with our best answer choice, answer choice A.

63) Like most standalone questions, this question relies almost exclusively on our knowledge of the content. Liver tissue has a high regenerative capacity; it’s often able to undergo extensive regeneration following injury or loss. This regeneration is due to cell division, and specifically mitosis. We want an answer choice that’s consistent with this.

    1. fission. This is something we find in prokaryotes and some organelles. Prokaryotes reproduce asexually by binary fission. Prokaryotes do not undergo mitosis; rather the chromosome is replicated, and the two resulting copies separate from one another, due to the growth of the cell. This answer choice doesn’t stand out for the time being. 
    2. meiosis. Meiosis is cell division in sexually-reproducing organisms used to produce the gametes, such as sperm or egg cells. The two nuclear divisions result in four nuclei that are not genetically identical and contain one chromosome set only that usually divide into four new haploid daughter cells. This is better than answer choice A, but cell regeneration corresponds to mitosis, not meiosis.
    3. mitosis. The purpose of mitosis is cell regeneration, growth, and asexual reproduction. In mitosis, a single nuclear division that results in two nuclei that are genetically identical to the original parent nucleus. In certain cells, like those of the liver in this question, mature cells remain capable of division to allow growth or regeneration after injury. We like this answer choice as the superior answer over Answer choices A and B.
    4. cell growth. Cell growth is an increase in cell volume or mass; it’s not the same thing as cell division which we covered with mitosis. Mitosis remains the superior answer choice. We’ll stick with our correct answer, answer choice C: Mitosis.

64) A few initial observations we can make here: Cyclin concentration is shown along the Y-axis, while we alternate between interphase and mitosis along the X-axis. X-axis shows parts of the cell cycle. 

Cyclin concentration increases during interphase, but cyclin concentration decreases during mitosis. That means we have a buildup of cyclin in interphase and cyclin is broken down during mitosis. We want an answer choice that is consistent with Figure 1 and these changes in concentration of cyclin. 

    1. Replication of the cyclin gene during S phase of interphase. While this could explain the increase in cyclin levels during interphase, we are not given any clues into why there is oscillation of cyclin concentration. Ideally, we get an answer choice that explains the fluctuation in cyclin concentration.
    2. Segregation of chromosomes carrying the cyclin genes during mitosis. This is similar to answer choice A because we don’t get any clear reasoning behind the rise and fall of cyclin concentration. We want an answer choice that explains the increase in cyclin concentration during interphase, and the decrease during mitosis.
    3. Translation of cyclin mRNA in interphase and proteolysis of cyclin protein in mitosis. This answer choice is consistent with what we’re looking for: Translation of cyclin mRNA in interphase corresponds to the increased cyclin concentration; proteolysis (the breakdown of protein) of cyclin protein in mitosis means decreased cyclin concentration. This is a superior answer choice to answer choices A and B.
    4. Translation of cyclin mRNA in mitosis and proteolysis of cyclin protein in interphase. This answer choice does explain why there are fluctuations in cyclin protein concentration, but the reasoning here is backwards. We have translation of cyclin mRNA in interphase. How do we know that? The increased cyclin concentration. We have proteolysis of cyclin in mitosis, and how do we know that? The breakdown of cyclin and the decreased cyclin concentration. We’re left with our best answer, answer choice C: Translation of cyclin mRNA in interphase and proteolysis of cyclin protein in mitosis.

Biology Question Pack Volume 1: Passage 10

65) The question stem explicitly says “According to the passage,” so in theory, we should answer this question using the information in the passage, and a breakdown of the experiment in the passage. From the passage, we know that by inhibiting bacterial synthesis and controlling the pH in the stomach, we can treat individuals presenting with stomach ulcers. With that being said, we likely also have to use general knowledge depending on the type of answer they’re looking for.

    1. Use of drugs that prevent the production of acid. This answer choice sounds like something that was explained in the passage itself. The passage mentioned that when drugs are used to neutralize or reduce the production of stomach acid, 95% of the patients have a recurrence of the ulcer within two years. Even though overproduction of hydrochloric acid may cause stomach ulcers, this alone doesn’t prevent recurrence.
    2. Use of drugs that neutralize stomach acid. Again, when drugs are used to neutralize or reduce the production of stomach acid, 95% of the patients have a recurrence of the ulcer within two years. This is virtually the same as answer choice A. Even though hydrochloric acid can cause ulcers, neutralizing stomach acid can be a factor in treatment, but not the entire treatment. 
    3. Use of drugs that inhibit bacterial protein synthesis. This sounds more like our prediction. Inhibiting bacterial protein synthesis will address the H. pylori issue. The passage says that when ulcer patients are treated with a combination of antibiotics and drugs that reduce acid production, there seems to be no recurrence of the ulcers. Even though this is a better answer choice than A or B, we still would like a combination of inhibiting bacterial protein synthesis and inhibiting acid production. We can keep answer choice C as the superior answer choice for now.
    4. Use of drugs that inhibit acid production and bacterial protein synthesis. Bingo, this addresses both of the causes mentioned in the passage, and it’s consistent with the treatment in the passage that led to no recurrence of ulcers. Answer choice C is incomplete, and Answer choice D is the most complete answer.

66) The passage says “H. Pylori was isolated from biopsies of ulcers; the organisms were found in greater than 95% of people with ulcers.” The also ties in something I talked about in the breakdown of Question 65 as well. I said that by inhibiting bacterial synthesis and controlling the pH in the stomach, we can treat individuals presenting with stomach ulcers. But for this question specifically, we’re focused on the cause of the stomach ulcers.

    1. people with stomach ulcers have antibodies to H. pylori. This is the opposite of what we would expect. If people with stomach ulcers had antibodies to H. Pylori, then they wouldn’t be adversely affected by the bacteria. 
    2. healthy individuals have antibodies to H. pylori. This answer doesn’t necessarily show that H. Pylori causes ulcers. This could be a strong correlation to provide evidence. In fact, this answer is probably better than answer choice A. But it still doesn’t show that H. pylori causes ulcers.
    3. ulcers could be produced in healthy organisms by infecting them with H. pylori. This answer is a good reason to believe H. pylori causes ulcers. By being able to infect a healthy organism with H. pylori, this shows that H. pylori actually causes ulcers. The key word here is cause. We can eliminate answer choices A and B, which do not show H pylori causes ulcers. Those are inferior answers to answer choice C.
    4. the organism can be passed from mother to fetus during pregnancy. Again, another answer that doesn’t effectively answer the specific question being asked. We’re talking about H. pylori causing ulcers, and this answer doesn’t support that. Answer choice C is correct here: ulcers could be produced in healthy organisms by infecting them with H. pylori

67) I’m going to reference the passage and specifically Figure 1, and explain what is happening in the figure. We can also take a quick peek at our answer choices before flipping back to the passage so we know what the question is asking for. We want an answer in terms of which strain grows fastest, and any resistance to streptomycin. 

We have our figure here, and we want to explain which strain grows fastest, and also reference the resistance to streptomycin. 

Look at the lines for Strain A (with the circles), and Strain A with streptomycin (with the squares). Adding streptomycin stunts the growth of Strain A. The untreated subculture grows much faster over time. The subculture treated with the antibiotic stagnates in terms of the number of cells. 

The lines for Strain B and the Strain B with streptomycin appear to be the same throughout the experiment. We see them overlap in the line that’s a triangle and X pattern. Strain B might be resistant to the antibiotic. Even after adding more streptomycin after 120 minutes, the two lines don’t deviate, and we don’t see a slowdown in growth in Strain B. 

Now compared to one another, we see the number of cells in the untreated Strain A rise much faster than the number of cells in the untreated strain B. That means Strain A grows more quickly than does Strain B.

    1. Strain A grows faster than Strain B. This is consistent with what I said during the breakdown of the question. I said Strain A grows faster than Strain B, and Strain B is resistant to streptomycin. 
    2. Strain A grows faster than Strain B and is also resistant to streptomycin. The first part of this answer choice is consistent with my breakdown, but Strain B is resistant to streptomycin, not Strain A. This contradicts what we saw in Figure 1
    3. Strain B grows faster than Strain A and is also resistant to streptomycin. The first part of answer choice C also contradicts Figure 1 and the breakdown of the question. Strain A grows faster, and Strain B is resistant to streptomycin. 
    4. Strain B grows slower than Strain A and is also resistant to streptomycin. Both parts of this answer choice are correct. Now we have to decide between answer choice A and D. The question stem asks, which best accounts for the results in Figure 1. The answer choice that explains which Strain grows faster, and correctly points out which Strain is resistant to streptomycin is going to be the best option here. We’re left with our correct answer, answer choice D: Strain B grows slower than Strain A and is also resistant to streptomycin.

68) We have another content question in disguise. As always, these can be tricky because they are located in a passage, which might mislead you into thinking you will need to use passage information to solve.

    1. This is correct. Metabolically, eukaryotes and prokaryotes carry out most of the same metabolic pathways, including glycolysis, anaerobic glycolysis, the citric acid cycle, and the electron transport chain. Where eukaryotes and prokaryotes differ is in the location of these pathways. In eukaryotes, glycolysis is located inside of the cytosol while the rest of the listed pathways are located somewhere in the mitochondria. In prokaryotes, all of these pathways occur in the cytosol. In terms of the electron transport chain, which is coupled to ATP synthase, the ETC & ATP Synthase of eukaryotes is located in the inner mitochondrial membrane, whereas the ETC & ATP Synthase of prokaryotes is located in the cell’s plasma membrane.
    2. Human cells and prokaryotes contain very chemically different ribosomes.
    3. Human cells do not have cell walls, unlike many bacteria.
    4. The shape of eukaryotic chromosomes is linear, whereas the shape of prokaryotic chromosomes is circular.

69) Once again, we’ll be focusing on causation and not correlation. We ultimately want to find a method to prove urease is necessary. One way we can do this is by trying to colonize the stomach when there is no urease present. If this is possible, then that invalidates the hypothesis. When we’re conducting a test like this, we want to start from an uninfected state.

    1. Exposing ulcer patients to antibodies to urease. In this case, the patients already have ulcers, so we are not effectively testing the colonization. We want to test in an organism that in uninfected to make sure we’re testing causation, not simply correlation. 
    2. Exposing uninfected animals to a strain of H. pylori that lacks urease. This answer choice would be able to effectively show whether urease is necessary for the colonization of the stomach by H. pylori. If the H. pylori are able to colonize, then urease is not necessary. We’re essentially able to disprove the hypothesis. If the animals remain uninfected, then that goes with the hypothesis presented. This is a superior answer choice to option A. 
    3. Exposing ulcer patients to radioactive urea. We want to test in an organism that in uninfected. Doing this would simply confirm there is H. pylori. Not test whether urease is necessary for colonization. This could be correlation, not causation. Answer choice B remains the best choice. 
    4. Measuring urease activity in biopsies of ulcers. Again, we want to test in an organism that in uninfected. This could once again be correlation, not causation. Just because there is urease activity, does not mean urease is necessary for the colonization of the stomach by H. pylori. Our best answer choice is answer choice B. The best test would be Exposing uninfected animals to a strain of H. pylori that lacks urease.

Biology Question Pack Volume 1: Passage 11

70) We’re given 3 different condition, and remember there may be more than one that we can pick. We can quickly reference the passage to see what causes rickets, and then we can go through each condition to see if it produces rickets. 

Here we have an excerpt from our passage that mentions rickets. It says rickets is “caused by insufficient vitamin D activity.” That means our answer is going to be caused to insufficient vitamin D activity. Key word here is caused.

We also know from the passage, and from our general knowledge, that vitamin D stimulates calcium absorption and enhances the effect of parathyroid hormone. Let’s not get tricked. Those are additional effects of vitamin D. If there’s insufficient vitamin D activity, that would cause a production of rickets, a decrease in calcium absorption, and a decrease in the effect of parathyroid hormone. Those 3 are correlated to one another. Only insufficient vitamin D activity is an exact cause.

Option 1 says metabolic deficiency of parathyroid hormone. We mentioned a lack of vitamin D would mean there’s not the same enhancement of the effect of parathyroid hormone. But this is not a direct cause of rickets. In fact, if parathyroid hormone was deficient, there would be a surplus of calcium because the bone matrix isn’t being broken down. That would mean higher mineralization, which is the opposite of rickets: Rickets corresponds to a lower ratio of mineral to organic matter. One thing I want you to note. Answer choices A through C all incorrectly list option I. What does that tell us? Our answer is likely going to be answer choice D. Answer choices A-C are all incorrect and contradict our prediction. We’re still going to go through our other answer choices for the sake of being thorough here though.

Option 2 says Impairment of conversion of vitamin D to its active form. This is consistent with our main cause of rickets. The passage explicitly tells us that vitamin D functions like a hormone when in its activated form. That’s when vitamin D activity is normal. If we impair the conversion of vitamin D to its active form, we won’t have proper vitamin D activity. So, option II is going to be a good choice. It’s also consistent with what we just said about answer choice D being our best option. 

Option 3 says Inability of the active form of vitamin D to act on its target tissue. This is similar to option 2. Even if vitamin D is active, if it can’t act on its target tissue, we don’t have the proper vitamin D activity. That means this is also a correct option. 

Going through our 3 options verified our initial prediction. We can stick with our correct answer, answer choice D (options II and III). We can eliminate answer choices A through C. Those all contradicted what we were told in the passage.

71) In other words, how does vitamin D play a role in calcium supplementation? We can reference the passage to find the relationship between calcium and vitamin D. Then use that information to answer our question. 

AAMC Biology question pack 1 difficulty

We have an excerpt from the passage below that talks about vitamin D, and specifically the relationship with calcium. We’re told vitamin D acts on the small intestine to stimulate absorption of calcium. What does that tell us? If we supplement with calcium, that might not increase our blood plasma calcium levels, or our body’s calcium levels in general. We need to actually be able to absorb the supplemented calcium. How does that happen? Through vitamin D acting on the small intestine to stimulate absorption.

    1. Vitamin D is needed to prevent rickets. This is a true statement, but does it answer the specific question being asked? This answer choice addresses the need for vitamin D in the body, but not in relation to calcium. We can call this answer out of scope, even though it’s not technically an incorrect statement. 
    2. The activated form of vitamin D stimulates the absorption of calcium into the blood. This answer choice actually relates vitamin D and the absorption of calcium. It’s also consistent with our prediction, and what we read in the passage. This explains the exact reason why both would want to be supplemented simultaneously. This is the superior answer so far.
    3. The activated form of vitamin D enhances the action of calcitonin. This answer choice presents us with a combination that we didn’t discuss in the passage, and is not relevant to the relationship between calcium and vitamin D specifically. We mentioned calcitonin in the passage as an important agent in fine tuning bone resorption. Not a direct contradiction, but we’re going to stick with our superior answer choice, answer choice B.
    4. The activated form of vitamin D enhances the uptake of calcium by bone tissue. This answer choice would be a great option, if it was correct in the context of this passage. The activated form of vitamin D acts on the small intestine to stimulate absorption of calcium. Not bone tissue. We’re left with our best answer, answer choice B: The activated form of vitamin D stimulates the absorption of calcium into the blood.

72) Once again, we’re given three different options and can pick more than one. We want to know which of the three options would increase, if there was a low level of calcium in the plasma. And said differently based on what was told to us in the passage: if we needed to increase bone resorption and increase plasma calcium levels.

Osteoclasts break down bone cells, and minerals, like calcium, are released. That allows blood plasma calcium levels to increase. This matches what we’re looking for, so we’re keeping option 1

Parathyroid hormone actually encourages the formation and activity of osteoclasts. We already established osteoclasts break down bone cells and allow for plasma calcium levels to increase. This is also a correct answer choice. If we glance at our answer choices, we see each answer choice has a maximum of two choices. So what does that tell us? We’ll still look at option 3, but unless it’s a clearly better answer, we have a viable answer with options 1 and 2.

Vitamin C. We said vitamin C is required for the synthesis of bone matrix, and bone formation. Bone formation is going to occur when there’s an overabundance of calcium in the blood, and calcium needs to be stored. This answer contradicts what’s asked of us in the question stem. 

We went through all of our answer choices already. I said options I and II are the only answers that correctly answer the question being asked. That means we can eliminate answer choices A, C, and D. Answer choice A is incomplete, and answer choice C and D both contain option 3 which contradicts what I said in my breakdown of the question. We’re left with the correct answer: answer choice B: I and II only

73) Based on what we read in the passage, calcitonin is present to counteract increased plasma calcium levels. Calcitonin decreases bone resorption, even though its effect is more a method of fine-tuning, not a significant effect. When would we have decreased bone resorption? When we don’t want additional calcium in the plasma, and when we want to increase bone formation. In other words, when calcium levels in the blood plasma are high. 

    1. When there is a dietary deficiency of calcium. This is the opposite of our prediction. A dietary deficiency of calcium would likely correspond to decreased levels of plasma calcium. Decreased plasma calcium would mean increased bone resorption, which is the opposite of calcitonin’s effect. 
    2. When there is a dietary deficiency of vitamin D. Vitamin D stimulates the absorption of calcium. That’s not needed when calcium levels in the plasma are already high. Vitamin D also enhances the effect of parathyroid hormone, which ultimately impairs new bone formation. That’s the opposite effect of what we want. We can’t eliminate either answer choice so far. 
    3. When the level of calcium in the plasma is high. This is consistent with our prediction. We have additional calcium in the plasma, and we want to increase bone formation. Calcitonin functions to decrease bone resorption when calcium levels in the plasma are high. This is the best answer so far.
    4. When the level of parathyroid hormone is too low. This is similar to answer choice B. Increased parathyroid hormone encourages the formation and activity of osteoclasts. In other words, breaking down bone cells and impairing bone formation. That happens when calcium levels in the plasma are too low-the opposite of what happens when the level of calcitonin tends to increase. We can eliminate answer choice D, so we’re left with our correct answer, answer choice C: The level of calcitonin tends to increase when the level of calcium in the plasma is high.

We did break down the question and each answer choice thoroughly. But what should we know from the content outline? Calcitonin levels increase when calcium in the plasma is high. Here’s a quick mnemonic to remember that by: CalcitonIN brings calcium IN to the bone. In other words, it promotes bone formation. We want to be thorough in breaking down each question, but knowing the content outline thoroughly also ends up saving us lots of time.

74) The parathyroid gland is the main receptor and control center for blood calcium levels. Essentially the gland detects when blood calcium levels are low, and functions to restore homeostasis. Removal of the parathyroid glands means we lose parathyroid hormone activity. Normally parathyroid hormone encourages the formation and activity of osteoclasts, and ultimately increases blood plasma calcium levels. This is no longer happening, so we’re assuming plasma calcium levels will be low.

    1. Severe neural and muscular problems due to deficiency of calcium in the plasma. This answer choice is consistent with our initial breakdown of the question. There’s going to be a deficiency of calcium in the plasma. Calcium is essential in muscle contraction. Ultimately calcium ions allow muscle cells to relax following contraction. Calcium’s also necessary in blood clotting and cellular communication. Without the parathyroid gland regulating blood calcium levels, we’d see a lot of issues. 
    2. An increase in calcitonin production to compensate for calcium deficiency in the plasma. The thyroid gland actually produces calcitonin. Calcitonin decreases bone resorption, which would further decrease calcium levels in the blood. This answer choice contradicts what I said in the breakdown of the question.
    3. A drastic change in the ratio of mineral to matrix tissue in bones. This answer choice is interesting, because it is possible. We’re not expecting the same level of bone resorption, meaning we would have a higher level of calcium in bones than usual. The ratio of mineral to matrix tissue itself shouldn’t change drastically though. There’s no reason to expect the synthesis of bone matrix to be adversely affected. We’ll stick with answer choice A as the more direct, and superior answer choice here.  
    4. Calcification of some organs due to accumulation of calcium in the plasma. This answer choice directly contradicts our prediction as well. We’re expecting a low level of calcium in the plasma, not an accumulation of calcium in the plasma. This is a direct contradiction. We’re left with our correct answer: answer choice A. Removal of the parathyroid glands would cause Severe neural and muscular problems due to deficiency of calcium in the plasma. 

A few notes: What was the focus of pretty much every question in this set? It was on the effect of bone resorption and bone reformation. We also tied that into calcium levels in the blood plasma and maintaining homeostasis. That was the big picture around which this passage revolved. Of course we had to know some details from our content, but everything tied back to the big picture. No need to get lost in the minutia during your first read-through of any passage. The big picture is where you’ll get a bulk of your points.

Biology Question Pack Volume 1: Passage 12

75) In other words, which of the 5 species given in our passage is least reliant on UV photoreceptors for communication? All of the lizards’ communication would theoretically suffer, but we want to know which would suffer the least. We’ll go back to Figure 1 in the passage, and we’re going to see the degree to which the dewlaps of the five species reflected UV light. 

In theory, the lizard that exhibited the smallest reflectance percentage would be least affected. Species E only has a reflectance percentage slightly above 10% at a wavelength of 360 nm. Why are we using that wavelength? The passage tells us this is the wavelength where we find UV light. Species E’s dewlaps are the least UV reflective, so species E relies on other means to visually communicate more heavily than the other species. If we eliminate UV photoreceptors in all of the species, species E would have to compensate the least. Species D, C, B, and A would all have to compensate more, in that order. 

We already have a predicted answer, but we also have to consider there are only 4 answer choices listed. Let’s quickly go through and make sure the answer we came up with is listed here, otherwise we’ll go with our 2nd choice. Our answer was just in reverse alphabetical order. 

Answer choice D matches our predicted answer. Species E would be disadvantaged the least if there were a mutation that eliminated UV photoreceptors. This question was essentially just analyzing the data given to us in Figure 1.

76) We’re asked about the position on a chromosome of the gene for UV reflectance pigment. We’re going to focus on genetics, and particularly gene position in sex chromosomes vs autosomes.

The locus of a gene is the position of the gene on a chromosome. The gene for UV reflectance pigment can be found in all lizards, even if the actual dewlap itself is only found in males. The question stem mentions all of the species of lizard had X-Y chromosomal sex determination. That’s the same as in humans. Males have both an X and a Y chromosome, females have X-chromosomes only. But again, even though males are the only ones with the dewlap, females can still have the pigment gene itself. We’re not told whether the gene is expressed or not.

    1. must be on the X chromosome. Like we mentioned in our initial breakdown, the locus of a gene for UV reflectance pigment can be found in all lizards, and can be found on a sex chromosome OR on an autosome, which is not a sex chromosome. Again, we may not see the gene expressed in female lizards because they don’t have a dewlap, we can’t rule out all lizards have the gene. That means we want to keep this answer choice for now.
    2. must be on the Y chromosome. This answer is more extreme than answer choice A. We mentioned only male lizards possess the Y-chromosome. But we also mentioned the pigment gene itself could be possessed by both males and females. Answer choice A is going to remain superior because it is less extreme, and less specific. 
    3. must be on an autosome. This answer choice is a plausible answer. The locus of the gene can be on an autosome as well. Autosomes are chromosomes that aren’t sex chromosomes. Again, we can find the locus on autosomes, even if we only see males express the gene through the dewlap. We’re going to keep answer choices A and C for now. 
    4. could be on a sex chromosome or on an autosome. This answer choice covers everything we mentioned so far. The locus of the gene can be found in both sex chromosomes, and autosomes. This is consistent with our breakdown of the question, and it’s inclusive of both answer choices A and C which we said were plausible answers. Answer choice D will be the best answer. Answer choices A-C are all too extreme and specific.

77) This comes straight from our content. The five species are closely related. But a species is a group of living organisms consisting of similar individuals, capable of exchanging genes or interbreeding. Even though the five species are closely related and are neighbors by proximity, we’re still going to remember they’re distinct species.

    1. one population inhabited the forest and the other lived in a field. This answer choice is not answering the specific question being asked. This option is addressing the location of the two species, not whether the lizards can exchange genes or interbreed. Even though the author made a generalization about the location of the 5 species, it’s possible for lizards of the same species to live in different locations. 
    2. one population had a UV-reflective dewlap and the other did not. This answer also does not address the definition of separate species, or the fact that these lizards may still be able to exchange genes or interbreed. Again, this answer doesn’t answer the specific question being asked about separate species. 
    3. they did not communicate with each other. Third straight answer that doesn’t address the exact definition of separate species. This is similar to options A and B. If we can’t find a superior answer choice, we may have to decide between all of these options to find the best answer choice.
    4. they did not interbreed and produce fertile offspring. This sounds almost verbatim like our prediction. We mentioned a species is a group of living organisms consisting of similar individuals, capable of exchanging genes or interbreeding. This was almost exclusively a content question. We can eliminate answer choices A-C, none of those correctly answered the question. Answer choice D is the superior answer. The lizard populations would be considered separate species if they did not interbreed and produce fertile offspring. 

78) We’re likely going to have to use the last part of our passage to revisit our results, but we’re going to have to use general knowledge to form a conclusion.

We have the last part of our passage here, and we want to make a conclusion about dewlap reflectance that’s supported by the passage. There’s a relationship between UV reflectance and habitat, and the quantitative data to support this is given in Figure 1. What do we see in Figure 1 around 360 nm where we’ll find UV light? The two species that were found in the darker habitat, species D and species E, have the lowest reflectance percentage. Species A-C had the unshaded field habitat and likely more sun, and we see a higher reflectance %. We can use that conclusion and our graph to go through the 4 answer choices.

    1. Lizard habitat is determined by dewlap reflectance for each species. This answer choice is giving us a causation, that may or may not be true. The lizards may have higher dewlap reflectance because of their habitat for example. We know there’s a correlation, but we’re not sure if there is causation. 
    2. High UV dewlap reflectance is most important in brightly lit habitats. This answer appears to be true when we look at the graph. The lizards in the highly lit habitats had high UV dewlap reflectance. There’s more light to reflect, so these lizards can be more reliant on communication through dewlap reflectance. The lizards in the forest have to rely on other means of communication as well, and less on dewlap reflectance. Let’s keep answer choice B as the superior answer. 
    3. High dewlap reflectance is most important in dimly lit habitats. Going by our breakdown of the question and Figure 1, this is the opposite of what’s true. High dewlap reflectance looks to be most important in brightly lit habitats. The lizards in the dimly lit habitats have the lowest UV dewlap reflectance, implying they’re the least reliant on dewlap reflectance as a means of communication. We can eliminate answer choice C, it’s a direct contradiction.
    4. Dewlap reflectance is highest at the blue end of the visible spectrum. Blue in the visible spectrum is typically found around 450 nanometers. Just glancing at Figure 1, we can see that 450 nanometers is not the highest point for any of the 5 species. We can eliminate this answer choice for contradicting Figure 1. We’re left with our correct answer, answer choice B: High UV dewlap reflectance is most important in brightly lit habitats

79) We can answer this question using our general knowledge of evolution, and natural selection. Even though this can be tangentially related to the passage, this could almost be a standalone question.

Natural selection is the reproduction of individuals with favorable traits. These individuals survive environmental change because of those traits. Natural selection is an inevitable outcome of three principles:

  1. most characteristics are inherited;
  2. more offspring are produced than can survive; 
  3. offspring with more favorable characteristics will survive and have more offspring than those individuals with less desirable traits.

We want to explain how dewlaps that reflect UV light would fit within this definition, and cause more offspring to be produced and survive.

    1. individuals with UV-reflective dewlaps produced more offspring than did individuals without them. This answer choice is consistent with the breakdown I did of the question. It ties into that definition of natural selection perfectly. 
    2. individuals with UV-reflective dewlaps were better able to communicate than individuals without them. This answer choice suggests a seemingly positive effect of the UV-reflective dewlaps, this doesn’t technically mean these individuals can produce more offspring, or survive at a higher clip. Answer choice A is the superior answer still.
    3. individuals with UV-reflective dewlaps were less subject to predation than individuals without them. This answer choice is better than answer choice B, but it still doesn’t address the need to produce offspring, and pass on favorable traits. It doesn’t exactly answer the question being asked, so answer choice A is still the best.
    4. individuals with UV-reflective dewlaps mated more frequently than did individuals without them. This is an interesting answer. Theoretically this could mean more offspring, but there’s no way to know. We don’t know if this mating actually led to more offspring. Option A explicitly says more offspring were produced. For that reason, we can eliminate answer choice D. Answer choice A is the superior answer choice here.

Biology Question Pack Volume 1: Questions 80-84

80) In other words, what does decreased aldosterone do to blood volume and blood pressure? Before we jump into our content, note something the test-maker mentions in the question stem: aldosterone stimulates sodium reabsorption by the kidneys. That means a deficiency in aldosterone would correspond to decreased sodium reabsorption; there is more sodium lost in the urine. This all comes from just reading the question stem, but we can also get into aldosterone and its role in the excretory system.

What does aldosterone do? Like the test-maker mentions, aldosterone causes the tubules of the kidneys to increase the reabsorption of sodium. It also causes an increase in the reabsorption of water into the blood. That means an increase in the volume of fluid in the body, which also corresponds to increased blood pressure. But remember, in this question we’re told we have a deficiency. That means our best answer is going to be the opposite. If we have an aldosterone deficiency, we expect a decrease in the volume of fluid in the body, and decreased blood pressure.

    1. Increased volume and increased pressure. That answer choice corresponds to something we would expect if we had increased aldosterone, not an aldosterone deficiency. We broke this down already, so not a great start here.
    2. Increased volume and decreased pressure. First part of this answer choice contradicts our breakdown. We expect decreased volume because aldosterone isn’t causing the increase in reabsorption of water. Even if there were increased blood volume, that would correspond to an increase in blood pressure. This answer choice is still half correct according to our breakdown, but we don’t like the logic here. Still, it’s better than answer choice A, where both parts of the answer contradicted our breakdown. Answer choice B is now our best option.
    3. Decreased volume and increased pressure. This is similar to answer choice B where half of this answer matches our breakdown, but we don’t expect an increase in blood pressure. If we have a decrease in blood volume, we expect decreased blood pressure, not increased blood pressure. We’ll hold on to this answer choice for now, along with answer choice B.
    4. Decreased volume and decreased pressure. This answer choice matches both parts of our breakdown, and the logic here makes sense also. Aldosterone deficiency means decrease in the volume of fluid in the body, and decreased blood pressure. If we have a decrease in blood volume, we expect a decrease in blood pressure also, so this matches what we know from our content. We can eliminate answer choices B and C: both were only partially correct, but both ultimately contradicted our content because of the reasoning for each answer. We’re left with our correct answer, Answer choice D: Decreased volume and decreased pressure.

81) Said slightly differently: which organs produce the hormones that regulate the human menstrual cycle? 

The female reproductive cycle, or menstrual cycle, is the physiological change that occurs in fertile women for sexual reproduction and fertilization. It’s divided into three stages: the follicular phase, ovulation, and the luteal phase. We know a bit about each phase from our general knowledge, but we want to focus on which organs are involved specifically. Let’s look at a breakdown of the three stages:

AAMC Biology question pack 1 difficulty

This comes straight from our science outline on our website, from content category 3B. If the writing here is too small, you should go on our website and pull up these images. I implore you to make sure you have this content down. It will make your life a lot easier.

Again, we’re focused on the organs involved. That’s as simple as going through each of the stages and noting what information we can pick out. Going through the figure and keeping our question in mind. We can say: The ovarian and menstrual cycles of female reproduction are regulated by hormones produced by the hypothalamus, pituitary, and ovaries. That includes the hypothalamic releasing factors, FSH and LH, and estrogen and progesterone. 

    1. Hypothalamus-thyroid-ovary. Two of the options here match our breakdown: hypothalamus and ovaries. Thyroid gland does not match our breakdown. The thyroid gland produces T3 and T4 hormones that affect the metabolic activity of the body’s cells. We’re still keeping this answer choice for now. It matches 2/3 of our breakdown which is a good start, but let’s see if we can get a more thorough answer choice.
    2. Hypothalamus-pituitary-ovary. This answer matches our breakdown. We said these are the 3 organs involved in the production of hormones that regulate the female menstrual cycle. That makes this a superior answer to answer choice A.
    3. Pituitary-thyroid-ovary. This is similar to answer choice A. We like 2 of these options, but again, thyroid is not a correct answer for us. That means we can go ahead and eliminate answer choice C. Answer choice B remains superior.
    4. Pituitary-adrenal glands-ovary. Another answer choice where we have two correct answers we already discussed: pituitary and ovaries. What about adrenal glands? Adrenal glands secrete epinephrine and norepinephrine, but those aren’t involved in the menstrual cycle. We can also eliminate this answer choice. We’re left with our correct answer, answer choice B: Hypothalamus-pituitary-ovary.

82) To answer this question, we’re going to focus on DNA replication being semi-conservative. According to the semiconservative replication model, the two original DNA strands will separate during replication. These two strands will serve as a template for a new DNA strand. That means each newly synthesized double helix is a combination of one old and one new DNA strand. We still have complementary pairs. The ratio of adenine and thymine to the ratio of guanine and cytosine are going to remain the exact same. The deoxynucleoside triphosphates themselves serve to build the new strands of DNA. The ratio of the dNTPs isn’t going to change the ratio in the daughter DNA molecule. That means the 3:1 ratio is going to remain exactly the same. We can pick the only answer choice that is consistent with that, which is answer choice D.

83) First thing we want to remember is RNA has the nucleic acid uracil, but DNA contains thymine. Next thing we want to recall is the location of transcription. In eukaryotes, that happens in the nucleus. DNA double helix unwinds near the gene being transcribed. Transcription uses one of the two exposed DNA strands as a template, and the RNA product is complementary to the template strand. That RNA product will almost be identical to the other DNA strand, except for one big difference we mentioned already. In the newly made RNA, all of the thymine nucleotides are replaced with uracil nucleotides.

    1. The nucleus. This answer choice is consistent with our breakdown. We said transcription takes place in the nucleus. That’s where we have uridine incorporated into RNA. We like this answer choice for now, but we still have to keep comparing to our other answer choices.  
    2. The Golgi bodies. Golgi bodies function to sort and package materials before they leave the cell. There are modifications of proteins and packaging, but we’re concerned with RNA synthesis and transcription. We’ll stick with our superior answer for the time being, answer choice A. 
    3. The ribosomes. We can actually break down answer choices C and D together because the two are related.
    4. The endoplasmic reticulum. There are free ribosomes, and there are ribosomes attached to the endoplasmic reticulum. RNA is needed for protein synthesis, but we’re focused on transcription, not translation. That means we can eliminate answer choices C and D. Neither one matches what we came up with during the breakdown of this question. We’re left with our correct answer, answer choice A: The nucleus.

84) We’re deciding which of the answer choices does not arise from the mesoderm during development. Key word is NOT, which means 3 answer choices are derived from embryonic mesoderm, and one is not. 

AAMC Biology question pack 1 difficulty

The mesoderm is one of the three tissue layers in the embryo. Like its name suggests, the mesoderm the middle tissue later and it develops into the circulatory system and connective tissues like bone, muscles, tendons, and the dermal layer. Make sure you review the figure above. There are lots of possible answers to this question, even within the broad breakdown we just did.

    1. Circulatory. This answer choice contradicts our breakdown and the above figure. We said the mesoderm develops into the circulatory system, meaning blood cells, vessels, and the heart. That means this answer choice is derived from embryonic mesoderm, and won’t be a good option for this specific question.
    2. Bone. This also contradicts our breakdown. We mentioned the mesoderm develops into connective tissues like bone. Just like answer choice A, this answer choice is derived from embryonic mesoderm, so not a great option. Neither one is more incorrect than the other, but neither is a good answer choice.
    3. Dermal. Third straight answer choice where the answer contradicts our prediction. This answer choice also falls under the connective tissue we mentioned in our prediction. The mesoderm develops into the dermal layer. Be careful to not get this confused with the epidermis. The ectoderm produces the epidermis.
    4. Nerve. Just like the ectoderm produces the epidermis, it also gives rise to the organs of the nervous system like the brain and spinal cord, and nerves. That means we like this answer choice. Remember, we’re finding an answer choice that is not derived from embryonic mesoderm. We already were okay eliminating answer choices A-C, those all contradicted our general knowledge. We’re left with our best answer, answer choice D: nerve.

Biology Question Pack Volume 1: Passage 13

85) We’re simply going to look at the differences between the bacteria and eukaryotic cells, and see which functions the bacteria could carry out.  We have aerobic heterotrophic bacteria enter primitive eukaryotic anaerobes. Aerobes require oxygen, while anaerobes may not require oxygen for metabolism. In other words, the bacteria utilize oxygen, while the primitive eukaryotic cells do not require oxygen. That’s the major difference we see on the surface, so we’re going to look for a function that requires oxygen.

    1. Glycolysis. Anaerobic glycolysis is not as efficient in terms of producing ATP, but is a method by which energy is produced in anaerobes. Both the aerobic bacteria and the eukaryotic anaerobes can carry out glycolysis. 
    2. Krebs cycle and electron transport. We’re going to look at this answer choice from the perspective of cellular respiration. The Krebs cycle and electron transport chain are both part of cellular respiration, which requires oxygen. The Krebs cycle would cease to continue if there were no oxygen to carry out the rest of cellular respiration. We also know oxygen is the final acceptor of electrons in the electron transport chain. This means answer choice B is a viable answer for a function in only the aerobic bacteria. Eliminate answer choice A now because B is a superior option.
    3. Cell division. Cell division is used to either reproduce, grow and develop, or repair. Both the bacteria and the eukaryotic cells would undergo cell division, regardless of being aerobes or anaerobes. Answer choice B is still the superior answer choice.
    4. Transcription and translation. Transcription and translation are similar to answer choice C. Both the aerobes and anaerobes will utilize transcription and translation to get from DNA to proteins. Our passage also mentions that mitochondria and present-day bacteria contain the components necessary for transcription and translation. These answers are not oxygen-dependent. The best answer still remains answer choice B: Krebs cycle and electron transport.

86) To answer this question, I want to address how proteins are made by mitochondria, and in the cytoplasm after being transcribed from nuclear genes. Scientists hypothesized mitochondria evolved from bacteria that entered primitive eukaryotic anaerobes. Even though most proteins in present-day mitochondria are made in the cytoplasm from mRNA transcribed from nuclear genes, the two are not mutually exclusive. Their symbiotic relationship has been evolving for millions (and probably billions) of years, so it makes sense that protein synthesis can change to a more efficient method. What’s a possible, more efficient method? Having proteins transcribed from nuclear genes instead of from mitochondria directly.

    1. Yes; the transfer of genes from symbionts to the eukaryotic nucleus could have occurred during the last billion years of evolution. This is a viable explanation, which is all we need to answer this question. Evolution over billions of years can lead to a lateral transfer of genes, just like is proposed in the question stem. Like I mentioned when I was breaking down the question, it’s possible the primitive eukaryotic host cell nucleus acquired some mitochondrial genes over these years. That includes genes for making mitochondrial proteins. 
    2. Yes; this difference from bacteria is unimportant, because the many similarities between bacteria and mitochondria provide sufficient evidence in favor of the hypothesis. This answer choice is unreasonable. If there is evidence against a theory, the theory has been disproven. This is in spite of the similarities. This answer choice is essentially changing the question, and changing the criteria by which the hypothesis is being tested. We can eliminate answer choice B. 
    3. No; the fact that mitochondrial proteins are made in the cytoplasm is convincing evidence that mitochondria do not have a bacterial origin. This answer choice is implying mitochondria don’t have a bacterial origin because the mitochondria are reliant on the host cell for protein synthesis. Bu mitochondria can make their own proteins. The passage actually mentions mitochondria contain tRNAs and ribosomes necessary for transcription and translation. This answer choice uses incorrect reasoning.
    4. No; because bacteria can make all their own proteins and mitochondria cannot, this disproves the hypothesis. This answer choice is addressing the question being asked, but the answer itself is factually incorrect. Again, we know mitochondria can make their own proteins. The passage actually mentions mitochondria contain tRNAs and ribosomes necessary for transcription and translation. We can eliminate answer choice D because it’s factually incorrect. Answer choice A is the best answer here.

87) We’re going to reference Figure 1, just like our question stem says. We’ll see what happens to ATP synthesis if protons can move across membranes.

We have Figure 1 from the passage here, and we’re told gramicidin inserts into membranes and creates a pathway for proton movement. We see protons being pumped out near the top of the image. We now have a higher concentration of hydrogen ions in the intermembrane space, and a lower concentration in the matrix. Normally the protons would want to reach equilibrium and diffuse back into the inner mitochondrial membrane. But, the protons can’t pass directly through the phospholipid bilayer of the membrane spontaneously. That’s where ATP synthetase came into the picture.

Normally protons move down the concentration gradient through the protein ATP synthetase. Hydrogen ions flow down their electrochemical gradient. ATP synthetase catalyzes the addition of a phosphate to ADP, and the formation of ATP.

Now alternatively, if we have an artificial pathway that allows for proton movement: we no longer than that gradient or stored energy. We’re not getting that same movement of protons down the concentration gradient and interaction with ATP synthetase. That ultimately leads to a decrease in ATP synthesis.

    1. increase, because of increased proton movement back into the mitochondria. The first part of this answer choice contradicts our prediction. We think ATP synthesis will decrease. This answer choice is assuming the presence of protons in the mitochondria will lead to more ATP synthesis. Rather, it’s the gradient and interaction with ATP synthetase that leads to additional ATP synthesis. 
    2. decrease, because of a decreased rate of hydrogen-atom donation by NADH. First part of this answer choice matches our prediction, but the reasoning is not reasonable. Allowing for proton movement would not decrease the rate of hydrogen donation by NADH. We’ll still keep answer choice B because it’s not a direct contradiction like answer choice A. We can eliminate answer choice A.
    3. decrease, because the proton gradient will rapidly reach equilibrium. This answer choice matches our prediction exactly. By allowing protons to move, we no longer have a gradient, and we instead reach equilibrium. ATP synthesis is reliant on that gradient, so we’d naturally see a decrease in ATP synthesis. This is a superior answer to answer choice B because of the explanation.
    4. not be altered, because sufficient protons will remain between the membranes to generate ATP. This answer choice is similar to answer choice A. The presence of more protons is not going to increase ATP synthesis if we don’t have that pH gradient. We won’t have the same interaction with ATP synthetase, and there will be a decrease in ATP synthesis. Answer choice C is the best answer. ATP synthesis would decrease, because the proton gradient will rapidly reach equilibrium.

88) To solve this question, I’m going to use my knowledge of bacteria, and see if it lines up with a potential characteristic of mitochondria. This is a very open-ended question, so I’ll analyze each answer choice one at a time and identify any characteristic of both bacteria and mitochondria.

    1. They should use 80S ribosomes. This answer choice lists a characteristic that isn’t present in bacteria. Bacteria have 70S ribosomes, not 80S ribosomes, so right away we’re not confident in this answer. 
    2. They should be incapable of binary fission. This is similar to answer choice A. This lists a characteristic that isn’t present in bacteria. Bacteria can reproduce by binary fission. DNA exists as a single, circular chromosome which is replicated. The copies separate from one another, and we have an inward pinch at the equator which results in two cells. Let’s hold on to answer choice B for now
    3. They should have circular DNA. This is the first answer than can be applied to bacteria. Mitochondria and bacteria both having circular DNA is a viable reason why the two are similar, while supporting the symbiotic hypothesis. We already said answer choices A and B contradicted what we know about bacteria. We’re now keeping answer choice C as the best answer choice so far. 
    4. They should be capable of anaerobic respiration. The hypothesis says mitochondria evolved from aerobic bacteria that entered symbiotic relationships with eukaryotic anaerobes. Is it necessary for mitochondria to be capable of anaerobic respiration? We mentioned this in an earlier, this could be an evolutionary change that’s taken place over the past billions of years. 

Both answer choices C and D are viable answers, and we’re looking for any option that’s characteristic of both mitochondria and bacteria. DNA contains important, basic genetic information. Mitochondria and bacteria having circular DNA is going to be important, just based on the significance of DNA. We likely don’t want to change anything DNA-related. Changing the method of respiration seems more likely. Answer choice D is clearly superior to answer choices A and B. But similarities in DNA trump similarities in respiration. We’re going to stick with our best answer, answer choice C. 

89) We’re going to reference Figure 1 once again. We’ll see what happens to ATP synthesis if we have potassium ions in the mitochondria.

We have Figure 1 from the passage here, and this is going to be very similar to Question 87. We’re going to have some changes, and decide what happens to ATP synthesis. We were given the steps in the electron transport chain in the passage, and we were told those events created both a pH gradient and an electrical potential across the membrane. Normally you have the positively charged protons in the intermembrane space on top here. That charge difference gives us that electrochemical gradient. If we have an influx of positively charged potassium ions, that electrochemical gradient no longer exists.

Normally, the gradient represents a stored form of energy, and can be used to make ATP with the help of ATP synthetase. Protons move down the gradient through the protein ATP synthetase. So these positively charged hydrogen ions flow down their electrochemical gradient. ATP synthetase catalyzes the addition of a phosphate to ADP, and the formation of ATP. 

If we don’t have that gradient, we can’t get that same movement of protons down the gradient and interaction with ATP synthetase. That leads to a decrease in ATP synthesis.

    1. decrease, because the K+ will compete with protons at the active site on ATP synthetase. First part of our answer choice is consistent with our prediction. We expect ATP synthesis to decrease. The reasoning here doesn’t match our prediction, or anything we discussed in the passage. There’s no evidence that the potassium cations are going to compete with protons at the active site on the enzyme.
    2. decrease, because movement of K+ into the mitochondrial compartments will disrupt proton movement into the intermembrane space. This answer choice is consistent with our prediction. We no longer have that gradient, like we just talked about. That means we won’t have the usual movement of protons. Disrupting this chain is going to decrease ATP synthesis, just like we predicted. We can keep answer choice B, and kill answer choice A. The reasoning given in answer choice B is more direct, and relevant to the question stem.
    3. increase, because the net positive charge in the mitochondria will cause increased movement of protons into the intermembrane space. First part of this answer choice contradicts our prediction, let’s look at the reasoning. Are we expecting increased movement of protons? We’re expecting the influx of potassium ions would influence our gradient and decrease the movement of protons. Answer choice C contradicts the passage, so we can eliminate it.
    4. increase, because the additional positive charge will further activate ATP synthetase. First part of this answer choice contradicts our prediction, let’s look at the reasoning. This is similar to answer choice A. We have no way of knowing if additional positive charge will further activate the enzyme. In fact, it’s more likely the enzyme will only bind the protons it normally binds. This answer choice also contradicted my initial breakdown of the question. That means we’re left with the correct answer: answer choice B. The rate of ATP synthesis in the mitochondria would decrease, because movement of K+ into the mitochondrial compartments will disrupt proton movement into the intermembrane space.

Biology Question Pack Volume 1: Passage 14

90) I can answer this question using what the author tells us in the passage about having the inverted X chromosome.

We have part of our passage here. There are a few things we want to focus on. First thing we’re going to notice, is the passage says An XiY male expresses the sex ratio trait: he sires only daughters. And next paragraph says the frequency of Xi is expected to increase to 100%. If we put these two sentences together, we can establish that all males will eventually be Xi genotypes. And that means only female offspring will be produced. If only female offspring are being produced, the species will eventually die out as there are fewer and fewer males with which to mate. 

    1. Extinction. This is consistent with our breakdown of the question and excerpt from the passage. Extinction might not happen right away, but eventually there will be no fertile males with which to reproduce. The population would get more and more female, and the population would not continue.
    2. Stable population size, with a predominance of females. This answer contradicts what we’re told in the passage. There will be a predominance of females, and we know males sire an equal number of offspring, regardless of standard or inverted X-chromosomes. Eventually a lack of males will cause the population to be distinct, not maintain a stable population size. We can eliminate answer choice B.
    3. Stable population size, with all individuals producing a 50:50 sex ratio. This is another contradiction. We’re expecting that as time passes, only female offspring will be produced. Population size will eventually decrease. We can eliminate answer choice C as well. Answer choice A is still our best option here.
    4. Stable population size, with some individuals producing an excess of females and some producing an excess of males. There are no individuals that will be producing an excess of males according to our passage. Population size will decrease, and ultimately the population will eventually die out. That means we’re left with our correction answer, answer choice A: Extinction.

91) We’re simply going to use math to solve for the proportion of offspring with different genotypes. We’re focused on XiY genotypes. We have males contributing a single Y chromosome, and we have females contributing an X chromosome.

    • 15% of our female population will contribute only Xi chromosomes. Those offspring will all be XiY
    • Of the next 50%, we’ll see half contributions of Xi, and half as Xs. Meaning 25% additional percent of the total population will be XiY.
    • Of the remaining 35% of the population, we have no contributions of the inverted X chromosome.

AAMC Biology question pack 1 difficulty

We combine our XiY genotype proportions. We have 15% + 25% + 0%. That gives us a total of 40%

This was a math problem where we did no rounding, and there are no units. Answer choice C says 40% and is consistent with our math.

92) We know any male drosophila that has the inverted X chromosome will have only daughters. That means every XiY fly will only pass the Xi chromosome onto its offspring. This also ties into the previous questions. As time progresses, there will be a predominance of females in the population. This is an open-ended question, so we’re jumping straight into our questions and using the passage if necessary.

    1. XiXs flies have the highest fitness of any genotype. This answer choice could be a good option, if we knew it were true. The passage does not mention that the heterozygous genotype has the highest fitness. 
    2. XiXi flies tend to migrate and introduce the Xi chromosome into new populations. Another assumption that was not supported by what we read in the passage. We’re not sure whether the flies that are homozygous for the inverted X chromosome migrate. 
    3. XiXi flies pass X chromosomes to all their offspring, but XsXs flies pass their X chromosomes to only half their offspring. This answer choice is factually incorrect. The female flies pass on their X-chromosomes, but only the males pass on all of their X-chromosomes to their daughters. Recall from our previous question, females pass on an X-chromosome, but there’s no guarantee that it will be the inverted chromosomes over the standard. Answer choice C directly contradicts the passage so we can eliminate it. We’re still comparing A and B to our remaining answer choice. 
    4. XiY flies pass their X chromosome to all their offspring, but XsY flies pass their X chromosome to only half their offspring. This answer choice is factually correct, and it’s referenced in the passage. We already know the XiY fly passes its X-chromosome to all its offspring, because there are only daughters. The male has to pass on an X-chromosome. The standard X-chromosome male does not always pass on the X-chromosome. We can keep answer choice D as the superior answer choice. Answer choices A and B both made assumptions that weren’t referenced in the passage.

93) First thing we want to notes is the original male was able to have both daughters and sons. That tells us right away his genotype is XsY. Why do we know that? Males with the genotype XiY can only have daughters. That eliminates half of our answer choices already.

Next, we’re told roughly half the sons sire only daughters, but the other half of the sons can sire both daughters and sons. That means half the sons have XiY genotypes, and the other half have XsY genotypes. That’s the only way this works. That means half of the sons got an inverted X-chromosome from the mother, and the other half got the standard X-chromosome. 

That means the father is XsY and the mother is heterozygous XiXs. 

AAMC Biology question pack 1 difficulty

We can kill eliminate choices B and D right away because we said XiY males can only have daughters. Now we’re left with answer choices A and C. We said the mother has to be heterozygous if her sons have the ability to sire sons. We’re left with our correct answer, answer choice A. Answer choices B-D are all incorrect.

94) We’ll use our passage to determine when the inverted X-chromosome might not reach 100% frequency. 

I’ve pulled up a small excerpt from the passage here. “If none of the Xi-bearing genotypes (XiY, XiXi, or XiXs) is selected against, then the frequency of Xi is expected to increase to 100%, unless other genes act to suppress expression of e and f.” Let’s focus on this sentence in the context of our question. We were told e and f genes are expressed, but we still don’t reach 100% frequency. That can only mean one thing at this point. We’re seeing Xi-bearing genotypes selected against. That’s the only other way we keep the frequency of the inverted X-chromosome from reaching 100%.

    1. XsXs flies have the lowest fitness of any genotype. This answer choice implies the opposite of what we expect to be true. If the homozygous standard X-chromosome flies have the lowest fitness of any genotype, we would still be able to reach 100% frequency. In fact, it would be easier. 
    2. XsXs flies have the highest fitness of any genotype. This answer choice is consistent with our prediction. The standard X-chromosome flies have the highest fitness, so they will remain in circulation. We’re effectively selecting against the Xi-bearing genotypes because the standard X-chromosome flies have the highest fitness. We can keep answer choice B for now. It’s superior to answer choice A which is the opposite of what we expect to be true.
    3. XiY flies and XsY flies have equal fitness. This contradicts our prediction and the passage. If these flies have equal fitness, we’re not selecting for, or against, any genotype. We said that was a requirement to not reach 100% frequency. 
    4. XiXs flies and XsXs flies have equal fitness. This answer is similar to answer choice C. It contradicts our breakdown of the question, and the passage. If the flies have equal fitness, we’re not selecting for, or against, any genotype. That was a requirement to not reach 100% frequency. We can eliminate answer choices C and D. We’re left with the correct answer: answer choice B XsXs flies have the highest fitness of any genotype.

Biology Question Pack Volume 1: Passage 15

95) This is 100% a content question. The question stem is essentially giving us the definition of an endocrine system hormone. The endocrine system produces and uses chemical signals: those are hormones. Hormones travel through the bloodstream and control the actions of cells and organs.

    1. Neurotransmitters. Neurotransmitters are chemicals that are released at synapses of neurons. There’s a transfer of the impulse to another nerve fiber, a muscle fiber, or another structure. We’re not dealing with chemical messengers traveling in the blood, which is what our question stem asks for. This answer choice doesn’t match the description of what our correct answer should be.
    2. Digestive enzymes. This answer choice is tying into the digestive tract portion of the question stem, but digestive enzymes are also not traveling through the bloodstream. Digestive enzymes are found in the digestive tract itself. Similar to answer choice A, this answer choice doesn’t match the description of what we’re looking for.
    3. Protein receptors. Protein receptors do not travel through the bloodstream. They’re molecules in target cells that receive signals. They are not the actual signal. We can eliminate answer choice C. Answer choices A and B are the superior answers, but not perfect.
    4. Hormones. This matches our prediction exactly. Endocrine glands produce and release hormones directly into the blood, just like our question stem mentions. These hormones are transported in the blood to the target organs like in the digestive tract. Hormones have high levels of specificity, meaning they react with certain target sites-such as the digestive tract and fat cells. Answer choice D is going to be our best answer here.

One thing I want you to notice. Right after reading the question, I pointed out that the question stem essentially gave us the definition of an endocrine system hormone. We could’ve picked the correct answer choice at that point. But I still like to go through each answer choice. If we didn’t make that connection right away, we can still eliminate answer choices A-C with reasoning. I ultimately get to answer choice D either way, and that’s our correct answer. 

96) These were the two hypotheses in the passage, so what did they seek to explain? No need to go back to the passage just yet for any details. One of the big things talked about in the passage is that several factors influenced body weight. That includes metabolism, behavior, genes, age, and environment. These all worked in conjunction, depending on the hypothesis, and influenced and determined body weight.

    1. How multiple, interacting factors determine body weight. This answer choice matches our prediction. We mentioned metabolism, behavior, genes, age, and environment all interact (depending on the hypothesis), and determine body weight. This answer choice answers our question as well because it applies to both hypotheses. 
    2. How individual factors acting alone influence body weight. This answer is implying that the factors that influence body weight all act independently. That’s not true. For example, the settling point hypothesis example given in the passage mentions an environment where high-calorie food is plentiful. That’s not enough to lead to obesity. The individual also needs a genetic predisposition to obesity. This contradicts our passage, so we can eliminate answer choice B.
    3. How metabolism and the environment influence body weight. This answer choice does not apply to both hypotheses. The settling point hypothesis mentions environment, but the set point hypothesis does not. That means this answer choice contradicts our passage. This is not also inclusive of every factor discussed in the two hypotheses. While this is a better answer than answer choice B, answer choice A is still the superior answer. 
    4. How the environment and behavior influence body weight. This is similar to answer choice C. It does not include all of the factors that influence body weight, meaning it’s incomplete. The two hypotheses use a variety of factors and explain how these factors determine body weight. We’re left with our correct answer, answer choice A. The two hypotheses serve to explain How multiple, interacting factors determine body weight.

97) In other words, which of the two hypotheses implies a person can intentionally change their baseline body weight? Similar to our last question, we don’t have to go back to our passage just yet. Rather we’re going to rely on the big picture talked about in the passage. No need for small details.

The set-point hypothesis says the body has an internal control mechanism, and body weight is maintained at a set point. That set point is predetermined. And diet and exercise can’t reset the set point over the long term.

The settling point hypothesis leaves room for flexibility, and we’re told weight can be stabilized at a new level because of environmental factors, and genetic predispositions. That means we want an answer choice that’s consistent with the idea that the settling point hypothesis allows a person to alter body maintenance weight.

    1. The set point hypothesis because a thermostat can be reset. This answer choice is taking the analogy from the set point hypothesis too literally. Even though a thermostat can be reset, the author mentions the set point is predetermined and doesn’t change. 
    2. The set point hypothesis because the set point can change with age. The reasoning of this answer choice is technically true, but it does not correctly answer the question being asked. Set point can change with age, but that is not deliberately altering body maintenance weight. Even though our breakdown said the correct answer is the settling point hypothesis, answer choice B is superior to answer choice A. We can eliminate answer choice A for being unreasonable and making a connection that is incorrect.
    3. The settling point hypothesis because, with the correct genotype, one’s metabolism may allow weight to stabilize at a new level. This answer choice starts out better than both answer choices A and B because the settling point hypothesis matches our prediction. We also mentioned that individuals with a genetic predisposition and the proper environment can alter body maintenance weight. We’ll keep answer choice C-it’s superior to answer choice B.
    4. The settling point hypothesis because diet and exercise cannot reset the set point. This answer choice almost contradicts itself. The question stem asks which hypothesis allows altering of body maintenance weight. The reasoning in the second half of this answer implies that set point can’t be altered. That’s a direct contradiction, and it also contradicts what the author mentions in the passage. We can eliminate answer choice D as well, so we’re left with our correct answer, answer choice C: The settling point hypothesis because, with the correct genotype, one’s metabolism may allow weight to stabilize at a new level.

98) This could easily be a standalone question; all we’re doing is recalling how glycogen is metabolized. Glycogen is broken down when glucose levels are low in the blood and the body needs glucose. That’s when glycogenolysis happens. Glycogen stored in the liver is broken down to form glucose. Alternatively, the liver also store glucose as glycogen when there is excess glucose in the blood.

    1. Stomach. This is an incorrect answer choice because it contradicts my initial breakdown of the question, and our general knowledge. The stomach is not involved in glycogen breakdown. Like I mentioned, this is essentially a standalone question that’s 100% reliant on knowing content. 
    2. Liver. This matches our prediction exactly. Not much else to say here. The liver regulates glucose levels in the blood and stores glycogen. When there’s a shortage of glucose, glycogen from the liver can be broken down. Let’s keep answer choice B, we can eliminate answer choice A.
    3. Pancreas. This is going to be the only tricky answer choice here. The pancreas does help regulate glucose levels through insulin and glucagon. Glucagon acts on the liver. But we mentioned glycogenolysis earlier, which is a liver pathway. Pancreas is likely going to be a good second-best answer, but liver is still superior. Let’s eliminate answer choice C.
    4. Small intestine. Glycogenolysis does not take place in the small intestine and isn’t directly influenced by the small intestine. Both Answer choices B and C are better answers than small intestine, but we’re sticking with our best answer, answer choice B: the Liver.

Biology Question Pack Volume 1: Questions 99-103

99) Which of the following changes would NOT interfere with the repeated transmission of an impulse at the vertebrate neuromuscular junction? To answer this question, we have to consider which changes will not interrupt a nerve impulse at the neuromuscular junction, or in other words, between a motor neuron and a muscle fiber. Key word here is not. We expect three of these answer choices will interfere with the repeated transmission of the impulse and one answer choice will not.

Neuromuscular junction is a chemical synapse between a motor neuron and a muscle fiber. Hence the name, neuromuscular. We’re focused on repeated transmission of an impulse at this junction, and muscle action. 

Let’s break down what happens. A motor neuron attaches to a myocyte at a motor end plate, forming a neuromuscular junction. The action potential of the neuron releases acetylcholine into the synaptic cleft. Because we’re dealing with muscle action, we also know we’re dealing with the neurotransmitter acetylcholine; that’s the major neurotransmitter in the parasympathetic division of the autonomic nervous system. The acetylcholine binds membrane-bound receptors on the motor end plate, which activates ion channels.

    1. Addition of a cholinesterase blocker. On the surface, this doesn’t match exactly what we discussed in our breakdown. A cholinesterase blocker means we don’t have the normal breakdown of acetylcholine in the synaptic cleft. We have excess acetylcholine, and we’d have continuous transmission. That also means receptors are no longer available to react to impulses, but rather we have continuous binding. 
    2. Addition of a toxin that blocks the release of acetylcholine. This answer choice directly contradicts our breakdown. If we block the release of acetylcholine, we would interfere with the transmission of an impulse at the junction. No presynaptic release of acetylcholine means we don’t have anything binding to the postsynaptic receptors. Answer choice A is actually a better answer choice than this. Answer choice A proposed continuous stimulation because of the presence of acetylcholine. Answer choice B is proposing no acetylcholine release whatsoever. Without acetylcholine we don’t get transmission of the impulse. That means answer choice A is superior.
    3. An increase in acetylcholine receptor sites on the motor end plate. Let’s break down what this answer choice would do. We have an increase in receptors, meaning acetylcholine can still bind, and we can still have repeated transmission at the neuromuscular junction. This is a good answer choice. Answer choice C is superior to both A and B.
    4. Addition of a substance that binds to acetylcholine receptor sites. This is similar to answer choice B, but instead of having no presynaptic release of acetylcholine, we don’t have any binding to receptor sites. Similar effect here, which means we can get rid of answer choice D as well. We’re left with our correct answer, answer choice C: An increase in acetylcholine receptor sites on the motor end plate.

100) Pay close attention here, we’re not asked where we find striated muscle. Skeletal muscle is also referred to as striated voluntary muscle, or skeletal striated muscle. Rather we’re asked about muscles with striated fibers specifically.

Three main types of muscle: skeletal, cardiac, and smooth. Only skeletal and cardiac muscle tissue has striated fibers. Skeletal muscle mostly describes muscles that attach to bones. Cardiac muscle is the muscle found on the walls of the heart. 

What does striated mean? That just means these muscle fibers will have a striated appearance under a microscope. There are long, fine fibers when you look at it closely. There are a lot of possibilities in terms of potential answers, but we did a good job of generally grouping the muscles here. Let’s jump into our specific options.

    1. the heart. This answer choice matches our breakdown of striated fibers. We said cardiac muscle is the muscle found on the walls of the heart. Cells of cardiac muscle, cardiomyocytes, appear striated under a microscope. The cells will have a single nucleus. Good answer so far! Let’s see if there is any competition for best answer.
    2. the uterus. Uterus contains smooth muscle, which we mentioned is not striated. It’s found in the digestive system, urinary system, and like we see in this answer choice, the reproductive system. There are no visible striations in smooth muscle, so we can eliminate answer choice B.
    3. arteries and veins. The majority of muscle that makes up the wall of blood vessels is vascular smooth muscle, so similar to answer choice B here. There are no visible striations in smooth muscle, so we can eliminate answer choice C.
    4. the small intestine. This is something we touched on in answer choice B. Smooth muscle is found in the digestive system, and in organs like the small intestine. That means we can also eliminate answer choice D. Answer choices B-D all have smooth muscle. We’re left with our correct answer, answer choice A: The primary muscle type in the heart has striated fibers.

101) The author gives us specific values that we’ll use to solve for net volume per minute.

The MCAT makes math questions a very specific way. The actual addition, subtraction, multiplication, and division isn’t hard. This isn’t testing how well you can do mental math. The test-maker wants to know if you can take the values given to you, and apply them properly. You’re going to find that you’ll likely be able to come up with any of the 4 answer choices listed if you manipulate the numbers in the question stem. That’s why it’s important to know exactly what the question’s asking us and what each number represents, so we can solve the question properly.

We’re solving for net volume per minute here. First thing we’re given is tidal volume. 800 mL per breath, means 800 mL is the volume of air delivered to the lungs with each breath taken, or each respiratory cycle. But we want to know the volume of fresh air that enters the alveoli per minute. We’re told we have anatomical dead space of 150 mL. That’s the volume of air that is not entering the alveoli and exchanging with blood. Since no gas exchange happens, it’s dead space, and has to be excluded from the net air entering the alveoli per minute. We can subtract the nonalveolar respiratory system volume, that 150 mL, from the tidal volume, 800 mL. That 650 mL represents the net volume of fresh air that enters the alveoli per breath. But that’s not what we’re looking for. We want to know net volume of fresh air that enters the alveoli each minute. We have 10 breaths per minute, multiplied by our net volume of 650 mL per breath. Note the units: breaths will cancel out. We’re left with 6500 mL per minute. Our answer choice needs to be a volume per minute, so we’re happy with our final units. 

We did no approximating, and no rounding here. We solved for an exact value, which corresponds to answer choice C. That means answer choices A, B, and D are all incorrect, but why would anyone incorrectly pick A, B, or D? This goes back to what I said prior to the breakdown of the question. The test-maker wants to know if you can take the values given to you, and apply them properly. Answer choice A involves dividing our net volume of 650 mL per breath by our breathing rate of 10 breaths per minute. That doesn’t give us the proper units, so that’s another reason we know this is an incorrect answer choice. Answer choice B involves adding tidal volume to the anatomical dead space volume of 150 mL, then dividing by breathing rate. This also yields incorrect units. And lastly, answer choice D involves incorrectly solving for net volume of fresh air entering the alveoli by using tidal volume and breathing rate first, but that’s followed by subtracting the anatomical dead space volume. Answer choice C is the best answer.

None of the math is particularly hard, like I mentioned. But the key here is to know exactly what the question’s asking us, converting our units properly, and knowing what each number represents. 

102) We’ll look into muscle contraction, and specifically the roles of actin and myosin. The process is extensive and has lots of details, so we’re going to do a quick summary before jumping into our answer choices, and we’re also going to use a visual to help us demonstrate what we’re looking for. 

AAMC Biology question pack 1 difficulty

Sarcomeres make up the contractile apparatus in skeletal muscle. Myosin and actin are protein filaments that make up sarcomeres. Myosin and actin filaments overlap, and slide past each other when muscles contract and relax. 

We’re focused on contraction, so when a muscle contracts: actin is pulled along myosin toward the center of the sarcomere. There’s shortening of the sarcomere. Meaning there’s pulling until the actin and myosin filaments are completely overlapped. Good overview for the time being. Make sure to review the figure as necessary and recap muscular contraction if you are shaky about anything.

    1. Both actin and myosin shorten, causing the muscle tissue to which they are attached to contract. For a muscle cell to contract, the sarcomere has to shorten, but the components of sarcomeres don’t shorten. Instead, actin and myosin filaments slide by one another, causing the sarcomere to shorten. The filaments remain the same length. 
    2. Both actin and myosin catalyze the reactions that result in muscle contraction. Actin and myosin don’t catalyze reactions. Instead, they’re involved in using ATP to generate force and movement. The movement we see happens because of binding of myosin to actin filaments. Myosin functions as a motor to drive the sliding filaments. This answer choice is also not accurately describing what happens in contraction, but we’ll keep both answer choices A and B for now because neither one is superior to the other.  
    3. Actin molecules are disassembled by myosin, leading to a shortening of muscle sarcomeres. Myosin binds to actin, and that ultimately causes a sliding of the two filaments relative to one another. ATP will subsequently bind myosin to detach it from actin, but there’s no disassembling going on. Another answer choice that’s not exactly true.
    4. Bridges between actin and myosin form, break, and re-form, leading to a shortening of muscle sarcomeres. This answer choice gets into the interaction of actin and myosin, and what I’ve touched on in my breakdown of the question and our 3 previous answer choices. Nerve impulses will stimulate the release of calcium ion. Increased calcium ion concentration will signal muscle contraction, first using two accessory proteins: troponin and tropomyosin. When there’s low calcium concentration, the troponin/tropomyosin complex will block interaction of actin and myosin-that means no contraction. Increased calcium concentration is when we have that bridge form between actin and myosin. We already talked about how that bridge will break with the binding of ATP. Subsequent reattachment and additional binding will ultimately cause the filaments to slide further. We have shortening of muscle sarcomeres. This is going to be our best answer choice here. Answer choices A-C were inconsistent with my breakdown of the question. We’re left with our correct answer, answer choice D: Bridges between actin and myosin form, break, and re-form, leading to a shortening of muscle sarcomeres.

103) First thing I want you to do is be crystal clear about the verbiage of this question. If the optic cup doesn’t develop in the embryo, then the lens also doesn’t form. Note the order here. We don’t have one area develop in the embryo (optic cup), and subsequently we also don’t have the lens form. That’s our evidence, but what exactly is this evidence for? That’s what we’re figuring out in this question. We’re trying to determine what it means when the optic cup fails to develop and also prevents the lens from forming.

You may or may not recall from your general knowledge, the optic cup developing will influence the production of the lens. In the context of this question, that makes complete sense. No optic cup means no lens, while normal optic cup means the lens forms. That ultimately means the optic cup is needed for lens production, and is possibly even the structure that induces lens production. Let’s find an answer choice consistent with our breakdown

    1. the process of neurulation follows gastrulation. This answer choice is out of scope. Remember, we’re going through 4 claims, and the evidence for these claims is the statement in the question stem. This claim has nothing to do with the optic cup being necessary, and possibly inducing lens production. There’s no clue that neurulation timing is dependent on the statement in the question stem. 
    2. the eye develops early in vertebrate morphogenesis. Another answer choice that may or may not be true, but it’s out of scope. In fact, we don’t fully care if it’s true or not in this situation, because that’s not what the author’s asking us. We’re being asked about a claim that can be supported by the evidence in the question stem. Eye development is not dependent on the fact that the optic cup is necessary, and possibly induces lens production. Two answers that are out of scope so far. 
    3. cells may induce neighboring cells to differentiate. This sounds like a claim we can get behind. If this claim is true, we need evidence. What might that evidence be? When the optic cup fails to develop in the embryo, the lens also fails to form. We need the optic cup to develop, and that induces the ectoderm to produce the lens. The evidence finally matches one of our claims. We can eliminate answer choices A and B. Answer choice C is the best option so far.
    4. cell differentiation is an “all or none” phenomenon. Cell differentiation can happen multiple times during the development in an organism. But let’s think of this from the perspective of our question specifically here. Does this statement have to do with the relationship between the development of the optic cup and the lens? It does not. So that means this answer is not only factually incorrect, but it also doesn’t answer the specific question being asked like answer choice C. We’re left with our correct answer, answer choice C: cells may induce neighboring cells to differentiate

Biology Question Pack Volume 1: Passage 16

104) If we know the mechanism, we can determine which action (or actions) are inhibited. Quick breakdown explaining ectopic pregnancies: it’s the development of a fertilized ovum outside the uterine cavity. Alternatively, in a normal pregnancy, the fertilized egg implants in the inner lining of the uterus. Why does this happen? Normally, fertilization of the ovum happens as the sperm swims up into the fallopian tubes. The embryo divides several times, and takes up to a week to reach the uterus. This doesn’t happen in ectopic pregnancy, so the drugs in the question stem will prevent the embryo from reaching the uterus.

    1. Contraction of the uterus. This answer choice is related to the tightening of uterine muscles that happens during pregnancy. This is not going to increase the movement of the ovum in the fallopian tube and get the ovum to the uterine cavity. 
    2. Secretion of follicle-stimulating hormone. If we inhibit the secretion of FSH, we won’t have the development of ova. If we inhibit FSH, we would more likely have infertility than ectopic pregnancy. We’ll still hold on to both answer choices A and B, but neither address the embryo not being able to reach the uterus in an ectopic pregnancy.
    3. Onset of menstruation. This answer choice is similar to answer choice B. If we inhibit menstruation, we’re essentially inhibiting pregnancy in general. This answer choice isn’t related to the risk factors that increase likelihood of ectopic pregnancies. 
    4. Transport of the ovum from ovary to uterus. This answer is consistent with our prediction, and the definition of ectopic pregnancy in the passage. The main issue in an ectopic pregnancy is implantation outside of the uterus. If a drug inhibits the transport of the ovum, the chances of implantation outside of the uterus increase significantly. The other answer choices did not correctly address or answer the question being asked. We can eliminate answer choices A-C, and pick our superior answer: Answer choice D.

105) We can use our passage, and specifically the symptoms and effects of ectopic pregnancy mentioned in paragraph 2.

This paragraph went over the negative physical effects of ectopic pregnancy. It says “the woman may experience lower abdominal discomfort and recurrent vaginal bleeding. As rupture of the tube occurs or becomes imminent, pain becomes severe, and the woman may collapse due to internal hemorrhaging.”

A little bit to unpack here. We have lower abdominal discomfort, bleeding, rupture of the fallopian tube, and internal hemorrhaging. Just going by these two sentences, the rupture and the hemorrhaging are the most severe. What is hemorrhaging? It’s severe bleeding. That sounds like a very serious issue that can ultimately be fatal.

    1. Answer choice A says severe hormonal imbalance. This answer choice could be an effect of ectopic pregnancy, but there’s no mention of hormonal imbalance being an effect of ectopic pregnancy, only a cause. Hormonal balances would have to be very severe to be fatal. 
    2. loss of blood when the fallopian tube ruptures. This is similar to our hemorrhaging prediction. We said there is severe bleeding following the rupture of the fallopian tube. That loss of blood can prove to be fatal if there is too much blood loss, too quickly. There may not be sufficient time to treat this loss of blood. Compared to answer choice A, answer choice B is a more direct answer that relates to the passage and the question being asked.
    3. infection in the region of the pregnancy. This is a very real problem, but it ties into answer choice B. Answer choice B is an emergency, while answer choice C is more likely to be able to be treated. The author mentions the affected segment of the fallopian tube can be removed, and blood can be drained. Infection can be treated similarly. This answer choice can prove fatal, but is more treatable than immediate, severe bleeding. We can eliminate answer choice C, simply because B is a superior answer.
    4. inadequate nutrition due to fetal use of maternal nutrients. This is a possible cause of death. But it’s neither mentioned in the passage, nor does this sound as sudden or extreme as hemorrhaging. We want to pick the most likely cause of death, not just possible causes of death. We can stick with our best answer, answer choice B: The most likely cause of death in these cases is loss of blood when the fallopian tube ruptures.

106) To answer this question, we’ll have to focus specifically on our knowledge of different reproductive hormones and their roles. Typically, an acute rise of luteinizing hormone (LH) triggers ovulation, and development of the corpus luteum. If we have delayed secretion of LH, that could delay ovulation. That would also delay stimulation of estradiol and progesterone production by the ovaries.

    1. Progesterone. Progesterone is a steroid hormone secreted by the ovaries. Secretion prepares the uterus for the implantation of a fertilized ovum and eventually pregnancy. Progesterone does not play a role in the start of ovulation. We mentioned in our prediction that LH will stimulate progesterone production. 
    2. Estrogen. Estrogen actually triggers a surge in the amount of LH produced, which eventually leads to ovulation. Estrogen is indirectly involved, so we can hold on to answer choice B for now. This answer choice is more directly tied to ovulation than answer choice A. Answer choice B is superior.
    3. HCG. We discussed HCG in our passage. HCG functions to properly maintain the corpus luteum and allow for the secretion of progesterone.  It’s crucial, especially in the early stages of pregnancy. That being said, we usually refer to HCG as the pregnancy hormone and use HCG levels to test for pregnancy, like we mentioned in the passage. HCG is not going to be the player responsible for timing ovulation. We can eliminate answer choice C for not answering the question correctly, and still keep answer choice B for now.
    4. Luteinizing hormone. This is the answer we’re looking for. When we go through AAMC’s content outline, the line we say for LH is something similar to our prediction: “LH is a hormone produced by gonadotropic cells in the anterior pituitary gland. In females, an acute rise of LH triggers ovulation, and development of the corpus luteum.” If we delay secretion of LH, we delay its action. Meaning we’d have delayed ovulation. We can keep answer choice D, and we can now eliminate answer choice B. Answer choice B triggers answer choice D. But ultimately, answer choice D, LH, is the hormone directly responsible for delayed ovulation.

107) We can go over the 3 main causes discussed in the passage, and we’ll find what they have in common. 

I’ve got paragraph 3 from our passage here. We have three causes mentioned in the first sentence here. It says “including abnormalities of the fallopian tube, the zygote, and the endocrine system.” The rest of the paragraph goes into detail of each, one by one. What’s the one thing they all have in common? Fallopian tube infections mean we see a partial block in the tube. The ovum doesn’t make it to the uterine cavity. Issues with the zygote mean premature attachment to the wall of the tube instead of the uterus. The ovum doesn’t make it to the uterine cavity, and if we have altered hormone levels that can decrease the motility of tubal cilia and inhibit ovum transport. Again, the ovum doesn’t make it to the uterine cavity. So our answer is going to mention the ovum not making it to the uterine cavity, and implanting elsewhere.

    1. Answer choice A says implant in the uterus. This answer choice is consistent with our prediction. There were three causes, and all of them led to the ovum not making it to the uterine cavity. 
    2. leave the ovary. The zygote does leave the ovary. It makes it to the fallopian tube, but it does not make it to the uterine cavity. This answer choice contradicts our passage, and our prediction. Answer choice A remains superior.
    3. reach the fallopian tube. This happens in every situation mentioned in the passage. Reaching the fallopian tube isn’t the issue. The issue is implanting in the fallopian tube, and not making it to the uterine cavity. We can eliminate answer choice C for contradicting our passage.
    4. begin its development. This is similar to answer choices B and C where the zygote does begin its development. In fact, the initial stages of an ectopic pregnancy are similar to those of a normal pregnancy according to our passage. We have the zygote leave the ovary, reach the fallopian tube, and begin its development. That means we’re left with our remaining, and correct answer. Answer choice A: Implant in the uterus.

Biology Question Pack Volume 1: Passage 17

108) We’ll have to explain the effects of vasodilation, and the effects it has on body temperature. First thing we want to do is note the environmental temperature. 33 degrees Celsius is 91 degrees Fahrenheit. If you’re not comfortable with Celsius, and you don’t know how to convert between the two, you should at least have a general sense of what some common temperatures are. Room temperature is about 70 degrees Fahrenheit and 21 degrees Celsius. Average body temperature is 98.6 degrees Fahrenheit, and 37 degrees Celsius. We’re dealing with an environmental temperature that’s between the two, but closer to average body temperature.

33 degrees is a relatively hot day, and we have vasodilation, meaning blood vessels become wider. That means blood travels faster, and the vessels themselves are now closer to the surface of the skin. What does that allow our bodies to do? Radiate heat into the environment. We give off heat. This is the opposite of what happens in cold temperatures when we have vasoconstriction, with temporary periods of dilation. That’s when you want to keep vessels away from the surface of the skin. Let’s use this information, and let’s go through our answer choices.

    1. slowing blood flow through the skin. This answer choice directly contradicts our breakdown of the question, and the definition of vasodilation. We’re expecting vessels to widen, meaning blood can flow more freely and quickly. 
    2. maintaining an even distribution of heat throughout the body. This answer choice is not the effect we want. Vasodilation allows for vessels to be closer to the surface of the skin, meaning humans can radiate heat into the environment. Vasodilation isn’t meant to maintain an even distribution of heat throughout the body. This answer choice is not a direct contradiction like answer choice A, so answer choice B is superior.
    3. radiating excess body heat into the environment. This answer choice is consistent with our initial breakdown. We mentioned vasodilation allows for excess body heat to be released into the environment. That allows humans to regulate body temperature in warmer temperatures. We can keep answer choice C as the superior answer to answer choice B. 
    4. preventing needed body heat from being lost to the environment. This answer choice is the opposite of what we want. We actually want to get rid of heat to the environment to account for the warm weather. This answer contradicts what the author says in the passage, and what we came up with in our breakdown of this situation. Vasodilation of cutaneous blood vessels allows humans to radiate excess body heat into the environment. We can eliminate answer choice D. We’re left with the correct answer: answer choice C. 

109) In other words, which organism will have the highest body temperature in a very hot environment? If you’re unsure how hot 45 degrees Celsius is, it’s 113 degrees Fahrenheit. Make sure you’re able to at least have a ballpark figure in your head for these conversions. Our answer is going to come from our general knowledge and thermoregulation, but this is a very open-ended question and there are countless organisms the author might mention here. The one thing we do want to note, is normal human body temperature is 37 degrees Celsius. 45 degrees Celsius is hotter than most cities will reach in a given year, so we’re dealing with a truly hot environment.

    1. Human. I mentioned in my breakdown of the question, humans have a normal body temperature of 37 degrees Celsius. How can body temperature be 37 degrees, even when environmental temperature is 45 degrees Celsius? We saw two ways in the passage: sweating and vasodilation.
    2. Kangaroo rat. This answer was one of the examples given in the passage of a vertebrate that has very concentrated urine. We didn’t get into any additional temperature regulatory mechanisms, so there’s not really a way to distinguish between answer choice A and B being a better option. Let’s keep comparing and see if we have any obvious answer choices that are better than both answer A and answer B. 
    3. Camel. This is also an example given in the passage of a vertebrate that has very concentrated urine. We didn’t get into any additional temperature regulatory mechanisms for camels either. We know answer choices B and C are better adapted for maintaining homeostasis, but we don’t have enough information to know which organism would have the highest body temperature. 
    4. Lizard. This answer choice is interesting because the author does mention reptiles in the passage. What did we say about the temperature regulatory mechanisms for reptiles? We said the adaptations for reptiles are more preventative. Lower metabolic rate, and less water loss are the major adaptations for reptiles. Reptiles aren’t vasodilating or sweating like humans, for example. So we’re expecting that if environmental temperature remains this hot for any prolonged period of time, the body temperature of the lizard would be the highest. The author doesn’t give us any methods by which the lizard can cool down. We can eliminate answer choices A-C, we’re left with our superior answer, and (often most importantly) the one discussed in the passage. Answer choice D: Lizard.

110) We want to relate dehydration with proper kidney function and any aspects of kidney function discussed in the passage.

I’ve added the 2nd paragraph from our passage. Let’s jump into the second sentence here, it says: “During dehydration, the kidneys may reduce their urinary output from the normal level of 1.0-1.5 L H2O/day to as little as 0.5 L H2O/day, and renal salt excretion may decline to near zero.”

We’re seeing the volume of filtrate moving through the kidneys drop to 1/3 of its normal level in some cases. Blood volume and blood pressure drop, and urine has to be much more concentrated to conserve water. The body can’t continue using sweating as a cooling mechanism because of the need to conserve water and maintain a minimum blood volume to allow for basic bodily functions like delivering oxygen to body tissues. This is all to say, there are lots of issues when we have severe dehydration. The kidneys especially have to deal with a much lower volume of filtrate, and much more concentrated urine.

    1. inadequate blood volume for effective filtration. This is certainly going to be a direct problem during severe dehydration. We already mentioned there would be a lower volume of filtrate. The kidney has to balance the pH of the blood, control blood pressure, and regulate osmotic pressure. But severe dehydration leads to inadequate blood volume for filtration, and we ultimately experience kidney failure.
    2. inability to produce sufficient urine. This answer choice is tangentially related to our prediction. The human body can produce much more concentrated urine when its dehydrated. The inability to produce sufficient urine has a bigger underlying problem, namely what we just touched on in answer choice A. This isn’t a contradiction, but this is not as good of an answer as answer choice A. 
    3. buildup of salts in the distal tubules. The movement and regulation of salts in the distal tubules is regulated by hormones. While this could be a potential side-effect, a buildup of salts is not a direct result of the severe dehydration. Severe dehydration would lead to inadequate volume for filtration first and foremost, which is answer choice A. Answer choice C would happen later, as a result of answer choice A. 
    4. increased body temperature. This answer choice doesn’t directly address the question being asked. Increased body temperature might occur when severely dehydrated, but that’s not going to cause kidney failure. That’s what we said we’re looking to answer here. Which of the answer choices causes kidney failure? Increased body temperature wouldn’t directly cause kidney failure like inadequate blood volume for effective filtration. We can elimintae answer choice D, it’s not listing a direct cause of kidney failure. We’re left with our correct answer, answer choice A: Kidney failure during severe dehydration is most likely due to inadequate blood volume for effective filtration.

111) First things first, tropical conditions correspond to warmer temperatures based on the verbiage used by the test-maker. We want to explain different ways by which the human body cools itself, and how heat stroke may happen in an individual that doesn’t sweat. If we have an individual with no sweat glands, the only other way they can cool themselves (according to the passage) is by circulatory adjustment, or vasodilation of cutaneous blood vessels. The fact that these individuals are likely to die of heat stroke in the tropics is a sign that sweating is pretty important. The passage mentions sweating is a type of evaporative cooling. Water and electrolytes are normally lost through sweat glands in the skin and this helps moisturize and cool the skin surface. But that’s not possible in these individuals. Rather they experience heat stroke because the only way they can cool themselves is through vasodilation. 

    1. gain, rather than lose, heat by evaporation. This answer choice mentions evaporation. We’re not expecting any evaporation to take place because these individuals do not have sweat glands. We do expect the human body may gain heat though, which can ultimately cause heat stroke.
    2. gain, rather than lose, heat by radiation. This answer choice is similar to answer choice A. It mentions gaining heat, rather than losing. Normally during vasodilation, we have vessels dilate and warm blood is closer to the skin. Heat can be lost via radiation. If the body were to gain heat by radiation instead, that would have the opposite effect as intended. And the ultimate result could be death by heat stroke. This is a viable option, and possible for individuals without sweat glands. We can eliminate answer choice A for mentioning evaporation which contradicts our breakdown of the question. We’ll keep answer choice B and keep comparing.
    3. need to use different mechanisms than in temperate zones to maintain body temperature. As we read through our passage, we only explored two mechanisms by which to maintain body temperature. Individuals without sweat glands would only be able to utilize one of these mechanisms. There’s no indication that individuals in tropical zones can utilize different mechanisms than the ones used by all other humans. Answer choice C is contradicting what we’re told in the passage about there being two methods by which to regulate body temperature in hot temperature.
    4. be better able to regulate body temperature than under temperate conditions. This answer choice contradicts what the question stem is asking us. If individuals in tropical conditions are better able to regulate body temperature, then there should be a decreased risk of dying of heat stroke. They should not be more likely. We can eliminate answer choice D. We’re left with our correct answer, answer choice B. Under tropical conditions, the human body may gain, rather than lose, heat by radiation.

Biology Question Pack Volume 1: Passage 18

112) We can flip back to our passage and see where the author mentions these two bacteria, and what potential problems are caused by their presence. A preliminary prediction we can use is that these bacteria produce protein toxins that are superantigens. These superantigens cause the release of many T cells and extreme levels of cytokines. Let’s take a look back at the passage.

Top paragraph I want to emphasize it says “Most studies of these pathogens have focused on the effects of the protein toxins they produce. Chemical and biological/immunological tests indicate that these toxins are superantigens.” I already discussed how superantigens can have a negative effect. We can also focus on the lower paragraph where the author says “This increased cytokine release is probably responsible for many of the acute problems seen in TSS.” That’s exactly what we’re looking for in this question. We have superantigens that activate 20,000 times as many T cells as normal. That, in turn, causes the release of massive levels of cytokines.

    1. multiplying to produce large numbers of bacteria. This answer choice does sound problematic, but has nothing to do with our breakdown of this specific question. The author pinpointed the exact method by which the bacteria cause problems in the passage, and it’s not related to the number of bacteria. This answer is out of scope-even if it were true, it’s not addressing the specific question being asked. 
    2. stimulating exaggerated immune responses. This sounds like our breakdown of the question. The author highlights increased cytokine release for being responsible for the acute problems seen in TSS. The author also mentioned 20,000 times as many T cells are activated as is normal. That sounds very exaggerated to me. That exaggerated response is what ultimately causes the issues in acute infections. We can keep answer choice B, and eliminate answer choice A. 
    3. causing autoimmune reactions. This is a result of the problems caused by the bacteria, but it’s not the cause of the autoimmune reactions themselves. The bacteria can cause exaggerated immune responses, which eventually can cause autoimmune reactions. But autoimmune means reactions to antigens produced by the body itself. Autoimmune diseases and reactions would not be from antigens from the two bacteria. This answer choice contradicts what we read in the passage. Answer choice B remains superior.
    4. inhibiting metabolic enzymes with toxins. This answer choice was never mentioned in the passage, or in our breakdown of the question. If this were mentioned and the author said this was true, we might have a different story. But in this case, we can stick with answer choice B which the author explicitly mentions in the passage. Answer choice B is the superior answer choice to answer choice D. The two bacteria cause problems by stimulating exaggerated immune responses.

113) The author mentioned high fever, hypotension, and a rash as symptoms and said TSS affects 3 organ systems. The rash shows the skin is affected. Hypotension is a sign the circulatory system is affected. The other things that are mentioned in this passage are the high fever and the issues with the immune system following the effects of superantigens. The lymphatic system is the site of many key immune system functions, so we’ll keep that in mind as a possible option. 

    1. The musculoskeletal system. The musculoskeletal system gives humans the ability to move using muscular and skeletal systems. It essentially provides form, support, stability, and movement to the body. Are any of these functions going to be directly affected by TSS? None of the symptoms mentioned in the passage directly affected this answer choice. 
    2. The digestive system. The digestive system breaks down food and works to absorb nutrients. This is similar to answer choice A. The digestive system might be affected indirectly, but we have no indication from the passage that the digestive system would be affected by TSS. We’ll hold on to answer choices A and B, but we’d like a better option if possible.
    3. The lymphatic system. We mentioned in our prediction that the lymphatic system is the site of many key immune system functions. What did we say was one of the big issues following infection by the two bacteria that cause TSS? Superantigens cause the immune system to not act normally. We have a much higher number of T lymphocytes, and the release of massive levels of cytokines. The lymphatic system has immune system functions, so it would be adversely affected by TSS. We can keep answer choice C for directly answering the question being asked with information provided by the author in the passage.
    4. The respiratory system. The respiratory system functions in thermoregulation, exchange of gases, and filtering material from inhaled air. Just like answer choices A and B, the respiratory system wasn’t explicitly mentioned in the passage. We also didn’t see any issues with any of the functions of the respiratory system. We can eliminate answer choice D as well. Answer choice C is the superior answer because it’s an answer that we gathered from what the author mentioned in the passage.

114) We want to know the ratio of activated T cells by superantigens versus conventional antigens. We’ll have to pull up part of the passage again to get some details.

Above I added the third paragraph from our passage here. We want to note that it talks about superantigens and says this “unique type of binding activates approximately 20% of the T lymphocytes, as opposed to 1 in 100,000 T cells activated by conventional antigenic stimulation.” Conventional antigenic stimulation leads to 1 in 100,000 T cells activated. Superantigens bind and activate 20% of T lymphocytes. So in the same 100,000 T cells, we’d have 20,000 activated. 

AAMC Biology question pack 1 difficulty

We can use our 20,000:1 ratio to pick the correct answer. This was a math problem with no rounding, approximation, or changing units. We can use our prediction to pick the correct answer, answer C: 20,000.

115) This is a simple math problem where we’ll compare the dosages needed of each strain to cause infection.

This is a simple division problem. We can put the higher exponent on top. 1 divided by 5 is 0.2. Exponent on our 10 is going to be 5 minus 4, or 1. Multiply out and we see the ratio of bacteria A to bacteria B is 2 to 1. We need twice as much of strain A for infection.

AAMC Biology question pack 1 difficulty

If you aren’t familiar with dealing with scientific notation, that’s completely fine. We can multiply out these numbers so they’re no longer in scientific notation. We have strain A that requires a dosage of 100,000 bacteria. Strain B requires 50,000 bacteria. We have the same ratio here when we simplify. Dosage ratio has to be 2 to 1, strain A to strain B. We still need twice as much of strain A for infection.

AAMC Biology question pack 1 difficulty

    1. Strain A is five times as potent as Strain B. This answer is simply an incorrect value. Strain A is actually less potent than strain B. And strain B is twice as potent, meaning we need twice as much of strain A for infection.
    2. Strain A is one-fifth as potent as Strain B. This answer choice correctly lists strain B as more potent. But strain B is twice as potent, not five times as potent. This is a better answer than answer choice A though, so we can eliminate answer choice A. We’re still looking for a better option if possible.
    3. Strain A is twice as potent as Strain B. This is similar to answer choice A. Strain B is more potent than strain A, not the other way around. 
    4. Strain A is half as potent as Strain B. This answer choice matches our math. We said we need twice as much of strain A for infection. That’s because strain A is only half as potent as strain B. Keep the correct answer: answer choice D.

Biology Question Pack Volume 1: Questions 116-120

116) We can think of this question a little differently. The gallbladder helps in digestion. If the gallbladder is not present, which answer choice could not be digested, and therefore not consumed as much? In other words, what role does the gallbladder play in digestion? The liver produces bile that is stored in the gall bladder. Main function of the gallbladder we’re going to say is storing bile produced by the liver. That bile is secreted in the small intestine. Bile helps in the digestion of fats, and absorption of fats in the small intestine.

    1. proteins. Removal of the gallbladder should not have an effect on the digestion of proteins. Chymotrypsin and trypsin will break down proteins, and those are produced in the pancreas, not the gallbladder. 
    2. polysaccharides. Removal of the gallbladder won’t prevent the digestion of polysaccharides, which actually begins in the mouth. Gallbladder doesn’t store or secrete amylases needed to break down polysaccharides. Neither A nor B are good answers so far. 
    3. triglycerides. The gallbladder stores and secretes bile, which is necessary in the digestion and absorption of fats. If a person does not have a gallbladder, they can’t digest fats as well, and are told to avoid high-fat diets for this reason. This is the only answer choice of the first three that relates to the main function of the gallbladder. Answer choice C is the best answer choice so far.
    4. lactose. Removing the gallbladder won’t typically affect the digestion of lactose. Traditionally, dairy can be high in fats, so lactose may be indirectly restricted, but this is still not our best answer choice. Removing the gallbladder does not have any direct effect on digestion of lactose. We’re left with our correct answer, answer choice C: Triglycerides. 

117) This is almost like a Jeopardy! style answer. We’re given a definition, and we have to supply the key term corresponding to that definition. We’re dealing with some of the eukaryotic cells’ most abundant proteins. The author specifically highlights that. Posttranslational modification of these proteins can affect the ability of the proteins to condense DNA. All we’re doing in this question is identifying which proteins the author describes in the question stem. We can use our general knowledge to focus on the mention of posttranslational modification, and condensing DNA.

First thing we can focus on are the 30 nanometer fibers. Histones will package and order DNA into 30 nm structural units. These units are called nucleosome complexes, and they can control the access of proteins to the DNA regions. 

But does this also tie into posttranslational modifications? Modifications are actually what act as signals, or tags, for histone proteins. These chemical modifications attach to amino acids in histone proteins, or to nucleotides in DNA. They affect how tightly wound DNA is around histone proteins. For example, methylation of DNA and histones causes more tight packing, and transcription factors can’t bind DNA. Histone acetylation results in loose packing, so transcription factors can bind DNA and genes are expressed. Everything in our question stem points to histones.

    1. tubulins. Tubulins are proteins, but that’s pretty much where the correlation to the definition in the question stem stops. Tubulins are globular proteins that make up microtubules. We can see a lot of post-translational modifications, but these modifications don’t affect the ability to condense DNA, because that’s not the function of tubulins.
    2. histones. This answer choice is what we were looking for from the beginning, and we’ve gone through histones at length. Histones are proteins, we see posttranslational modifications like methylation and acetylation, and histones will package and order DNA into 30 nanometer structural units. This answer choice checks all of our boxes. It’s also superior to answer choice A.
    3. transcription activators. Transcription activators are proteins. They increase transcription of a gene or set of genes, but they’re not responsible for condensing DNA into 30-nanometer fibers. Post-translational modifications can actually regulate the activity of these activators. Still, we’ll stick with our best answer so far, answer choice B. The function of transcription activators contradicts what we’re looking for in the question stem.
    4. DNA polymerase subunits. DNA polymerases catalyze the synthesis of DNA molecules. Once again, the function we’re looking for is the ability to condense DNA into 30 nanometer fibers, which isn’t the job of DNA polymerase subunits. We can eliminate this answer choice for not matching the criteria in our question stem. That means we’re left with our best answer choice, answer choice B: Histones.

118) An X-ray of the bones of a leg means we’re dealing with long bones. Long bones are bones that are longer than they are wide, so good chance that’s what we have in the leg. What distinguishing quality of a growing child’s leg bones will show up on an X-ray? In other words, how can a doctor distinguish between a growing child’s leg bones, and an adult’s leg bones? 

In growing, child bones we see epiphyseal plates at the ends of long bones. These plates are just the area of growth in a long bone. During development, bones get longer at epiphyseal plates by conversion of excess cartilage to bone, through ossification. Ossification, is just the process of bone formation. That means we expect cartilage near the ends in a growing child. We’d have that cartilage be fully converted to bone in adults, where we wouldn’t see cartilage in the X-ray. 

    1. cartilaginous areas in the long bones. This answer choice matches our breakdown. We mentioned that during development, bones get longer at epiphyseal plates. That happens by conversion of excess cartilage to bone, through ossification. That means in a child, we’d still have that cartilage present. We like this answer choice for now.
    2. bone cells that are actively dividing. This answer choice presents an option that’s present in a growing child. But is this specifically what we’re looking for? Think back to the question stem. We want to know: how can a doctor distinguish between a growing child’s leg bones, and an adult’s leg bones? Dividing bone cells are doing to be found in both children, and adults. We can eliminate answer choice B. 
    3. the presence of haversian cells. I believe that should be canals*. These canals are essentially tubes that contain blood vessels and nerves. But this is similar to answer choice B. We’re not going to find these in only a growing child’s leg bones, or only an adult’s leg bones. Haversian canals are found in both. Answer choice A still remains superior.
    4. shorter-than-average bones. This answer choice is too extreme. While it might be true that children generally have shorter bones, that’s not a surefire way to distinguish between a growing child’s bone and an adult’s bone. It’s not always true a child will have shorter than average bones. Just like it’s not always true an adult will have average, or longer than average bones. We’re left with our correct answer, answer choice A: cartilaginous areas in the long bones.

119) First thing we want to remember, RNA has the nucleic acid uracil, but DNA contains thymine. Next thing we want to recall is where uridine nucleotides are incorporated into RNA. In other words, the location of transcription. In eukaryotes, that happens in the nucleus. DNA double helix unwinds near the gene being transcribed. Transcription uses one of the two exposed DNA strands as a template, and the RNA product is complementary to the template strand. That RNA product will almost be identical to the other DNA strand, except for one big difference we mentioned already. In the newly made RNA, all of the thymine nucleotides are replaced with uracil nucleotides.

    1. Nucleus. This answer choice is consistent with our breakdown. We said transcription takes place in the nucleus. That’s where we have uridine incorporated into RNA. We like this answer choice for now, but we still have to keep comparing to our other answer choices.  
    2. Lysosome. Lysosomes contain digestive enzymes for breaking down parts of the cell, and material taken into the cell by phagocytosis. Lysosomes are found in eukaryotic cells, but that’s the extent of where this could be a correct answer. Lysosomes aren’t involved inn transcription, so we can eliminate this answer choice.
    3. Ribosome. Translation occurs in ribosomes, which are like a factory for protein synthesis. We know RNA is needed for protein synthesis, but we’re focused on transcription, not translation. The process of incorporating uridine nucleotides into nucleic acid polymers doesn’t take place in ribosomes. We can also eliminate answer choice C.
    4. Golgi body. Golgi bodies function to sort and package materials before they leave the cell. There are modifications of proteins and packaging, but we’re concerned with RNA synthesis and transcription. We’re focused specifically on incorporating uridine nucleotides into nucleic acid polymers. We can eliminate answer choice D and stick with our best answer, answer choice A.

120) Think of a very hot day in high humidity. What happens? We sweat! But alternatively, when we go into a swimming pool or a bath, why don’t we absorb all of the water? That’s what we’re explaining here. We don’t need to focus too much on the specific makeup of the epidermis. We’re more focused on why we’re able to sweat.

We’ll start with sweating. Water and electrolytes are lost through sweat glands in the skin. That sweat has a job. It helps moisturize and cool the skin surface. Sweating is used primarily to regulate body temperature. But it assists the assists excretory system by removing waste.

Main takeaway here is: sweating is the production of fluids secreted by the sweat glands in the skin. Sweat is mostly water and sodium chloride, but it’s not coming out of passageways that go into our body. Rather sweat is secreted through channels that are continuous with the epidermis. Sweat comes out through pores, but water from the outside of the body cannot use these pores as a way into our bodies.

    1. the salt in sweat allows it to diffuse through the skin. This answer choice is trying to be a little tricky. It starts by mentioning there’s salt in sweat. That part of the answer choice is true, but does that salt allow sweat to diffuse through the skin? Not at all. The author explicitly tells us the epidermis is relatively impermeable to water. We’re literally told that in the question stem itself. Salt does not make the epidermis permeable to water.
    2. sweat glands have special channels through the skin. This matches our breakdown. We said sweat is secreted through channels that are continuous with the epidermis. Water is lost through sweat glands, and these pores are not a way into our bodies. This answer choice is consistent with our breakdown and it’s superior to answer choice A.
    3. an osmotic gradient in sweat moves it through the skin. This answer choice offers an explanation, but doesn’t address the barrier we’re presented in the question stem. Sweat doesn’t move through the skin, because the epidermis is impermeable to water. Instead, water and electrolytes are lost through sweat glands in the skin. We don’t see sweat move because of an osmotic gradient. Answer choice B remains the best option.
    4. sweating occurs in only those areas of the body where the skin is water permeable. This answer choice is unreasonable. Think of when we visualized the hot day. We said when it’s a hot day we tend to sweat. But when we jump into a pool, we’re not constantly absorbing water. Why is that? Because the epidermis is relatively impermeable to water. When we lose water from the body, we’re losing it from sweat glands and pores. Water doesn’t come into our bodies in the same area. We’re left with our correct answer, answer choice B: sweat glands have special channels through the skin.

What is a good percentage for AAMC question packs?

Question packs are just content. They aren't like the exam. That being said, all I know is 80-90%+ for a 520+. The higher the better.

Do AAMC question packs help?

The science Question Packs are good material, but because they're not as representative of the new test, they're not essential. If you were running out of time before your test and had to skip one piece of AAMC material, I'd recommend that you skip the science Question Packs.

Is AAMC bio Qpack 2 difficulty?

The Question Packs tend to be on the easy side. The wording and structure of the questions is very similar to the real test, but there are a higher proportion of knowledge-based questions vs. comprehension questions than on the real test.

How many AAMC question packs are there?

The online AAMC MCAT® Official Question Pack Bundle includes all six question packs, with 720 unique passage-based and independent questions that allow you to focus on Biology, Physics, Chemistry, and Critical Analysis and Reasoning Skills.